Sei sulla pagina 1di 165

ENGINEERING MATHEMATICS-I

15MAT11

SYLLABUS
Engineering Mathematics-I
Subject Code: 15MAT11
IA Marks: 20
Hours/Week: 04
Exam. Hours: 03
Total Hours: 50
Exam. Marks: 80
Course Objectives
To enable students to apply knowledge of Mathematics in various
engineering fields by making hem to learn the following:
nth derivatives of product of two functions and polar curves.
Partial derivatives.
Vectors calculus.
Reduction formulae of integration to solve First order
differential equations
Solution of system of equations and quadratic forms.
Module 1
Differential Calculus -1:
Determination of nth order derivatives of Standard functions Problems. Leibnitzs theorem (without proof) - problems.
Polar Curves - angle between the radius vector and tangent, angle
between two curves, Pedal equation for polar curves. Derivative of
arc length - Cartesian, Parametric and Polar forms (without proof)
- problems. Curvature and Radius of Curvature Cartesian,
Parametric, Polar and Pedal forms(without proof) and problems.
10hrs
Module 2
Differential Calculus -2
Taylors and Maclaurins theorems for function of o ne
variable(statement only)- problems. Evaluation of Indeterminate
forms.
Partial derivatives Definition and simple problems, Eulers
theorem(without proof) problems, total derivatives, partial
DEPT OF MATHS, SJBIT

Page 1

ENGINEERING MATHEMATICS-I

15MAT11

differentiation of composite functions-problems, Jacobiansdefinition and problems .


10hrs
Module 3
Vector Calculus:
Derivative of vector valued functions, Velocity, Acceleration and
related problems, Scalar and Vector point functions.Definition
Gradient, Divergence, Curl- problems . Solenoidal and Irrotational
vector fields. Vector identities - div ( F A), curl ( F A),curl (grad F
), div (curl A).
10hrs
Module- 4
Integral Calculus:
Reduction formulae sinnx dx cosnx dx sinnxcosmxdx,, (m and n
are positive integers), evaluation of these integrals with standard
limits (0 to /2) and problems.
Differential Equations:
Solution of first order and first degree differential equations
Exact, reducible to exact and Bernoullis differential equations.
Applications- orthogonal trajectories in Cartesian and polar forms.
Simple problems on Newtons law of cooling..
10hrs
Module 5
Linear Algebra Rank of a matrix by elementary transformations,
solution of system of linear equations - Gauss- elimination method,
Gauss- Jordan method and Gauss-Seidel method. Rayleighs
power method to find the largest Eigen value and the
corresponding Eigen vector. Linear transformation, diagonalisation
of a square matrix, Quadratic forms, reduction to Canonical form
10hrs

DEPT OF MATHS, SJBIT

Page 2

ENGINEERING MATHEMATICS-I

15MAT11

CONTENTS
Module I : Differential Calculus- I.4 - 48
Module II : Differential Calculus- II49 -81

Module III: Vector Calculus........82-104

Module IV : Integral Calculus105-125

Module V : Linear Algebra 126-165

DEPT OF MATHS, SJBIT

Page 3

ENGINEERING MATHEMATICS-I

15MAT11

MODULE I
DIFFERENTIAL CALCULUS-I
CONTENTS:
Successive differentiation ..3
nth derivatives of some standard functions...7
Leibnitzs theorem (without proof)..16
Polar curves
Angle between Polar curves.20
Pedal equation for Polar curves...24
Derivative of arc length.28
Radius of Curvature.34
Expression for radius of curvature in case of Cartesian Curve 35
Expression for radius of curvature in case of Parametric
Curve..36
Expression for radius of curvature in case of Polar Curve...41
Expression for radius of curvature in case of Pedal Curve...43

DEPT OF MATHS, SJBIT

Page 4

ENGINEERING MATHEMATICS-I

15MAT11

SUCCESSIVE DIFFERENTIATION
In this lesson, the idea of differential coefficient of a function and its successive
derivatives will be discussed. Also, the computation of nth derivatives of some
standard functions is presented through typical worked examples.
1.

Introduction:- Differential calculus (DC) deals with problem of calculating rates of


change. When we have a formula for the distance that a moving body covers as a
function of time, DC gives us the formulas for calculating the bodys velocity and
acceleration at any instant.
Definition of derivative of a function y = f(x):-

f ( x x) f ( x)
x
The derivative of a function y = f(x) is the function f (x) whose value at each x is
defined as
dy
= f (x) = Slope of the line PQ (See Fig.1)
dx
f ( x x) f ( x)
= lim
-------- (1)
x 0
x
= lim (Average rate change)

Fig.1. Slope of the line PQ is

x 0

= Instantaneous rate of change of f at x provided the limit exists.


The instantaneous velocity and acceleration of a body (moving along a line) at any instant
x is the derivative of its position co-ordinate y = f(x) w.r.t x, i.e.,
dy
Velocity =
= f (x)
--------- (2)
dx
And the corresponding acceleration is given by
Acceleration

d2y
f ( x)
dx 2

DEPT OF MATHS, SJBIT

---------- (3)
Page 5

ENGINEERING MATHEMATICS-I

15MAT11

Successive Differentiation:The process of differentiating a given function again and again is called as
Successive differentiation and the results of such differentiation are called
successive derivatives.
The higher order differential coefficients will occur more frequently in spreading
a function all fields of scientific and engineering applications.
Notations:

dny
dy d 2 y d 3 y
, 2 , 3 ,.., nth order derivative:
dx dx dx
dx n
ii f (x) , f (x) , f (x) ,..., nth order derivative: f n (x)
iii Dy, D 2 y , D 3 y ,..., nth order derivative: D n y
iv y , y , y ,, nth order derivative: y (n )
v. y1 , y 2 , y 3 , nth order derivative: y n
i.

Successive differentiation A flow diagram

Input function: y f (x)

Operation
d

derivative)
Input function y f (x)
derivative)
Input function y f (x)
derivative)

df
f (x)
dx

(first order

Output function y

d2 f
f ( x) (second order
dx 2

Output function y

d3 f
f ( x) (third order
dx 3

dx

Operation
d

dx

Operation
d

Output function

dx

-----------------------------------------------------------------------------------------------------------Input function y n1 f n1 ( x)

Operation
d

dx

Output function y n

dn f
f n (x) (nth order
n
dx

derivative)

DEPT OF MATHS, SJBIT

Page 6

ENGINEERING MATHEMATICS-I

15MAT11

Calculation of nth derivatives of some standard functions


Below, we present a table of nth order derivatives of some standard functions for
ready reference.
Sl.
No

y = f(x)

1
2

e mx
a mx

ax bm

dny
yn n D n y
dx
n mx
m e
n
m n log a a mx
i.
ii.
iii.
iv.

1
ax b

5.

mm 1m 2....m n 1a n ax b
0 if m n
n! a n if m n
m!
x mn if m n
m n !

mn

for all m .

(1) n n ! n
a
(ax b) n 1
(1) n (m n 1) ! n
a
(m 1) !(ax b) m n

ax b m

6.

log( ax b)

(1) n 1 (n 1) ! n
a
(ax b) n

7.

sin(ax b)

8.

cos(ax b)

a n sin(ax b n )
2

9.

e ax sin(bx c)

10.

e ax cos(bx c)

a n cos(ax b n )
2
r n e ax sin(bx c n ) , r a 2 b 2

tan 1 ( b a)

r n e ax cos(bx c n ) , r a 2 b 2
tan 1 ( b a)

We proceed to illustrate the proof of some of the above results, as only the
above functions are able to produce a sequential change from one derivative to
the other. Hence, in general we cannot obtain readymade formula for nth
derivative of functions other than the above.

1. Consider e mx . Let y e mx . Differentiating w.r.t x, we get


y1 memx . Again differentiating w.r.t x, we get
y 2 m me mx = m 2e mx
Similarly, we get
y3 = m 3 e mx

y4 = m 4e mx
.
And hence we get

DEPT OF MATHS, SJBIT

Page 7

ENGINEERING MATHEMATICS-I
y n = m n e mx
2.

15MAT11

d n mx
e m n e mx .
n
dx

ax bm
m
let y ax b Differentiating w.r.t x,
m 1
y1 = m ax b a . Again differentiating w.r.t x, we get
m 2
y2 = m m 1 ax b a 2
Similarly, we get
m3
y3 = m m 1 m 2 ax b a 3
.
And hence we get
mn
y n = m m 1 m 2 . m n 1 ax b a n for all m.
If m n (m-positive integer),then the above expression becomes
nn
y n = n n 1 n 2 .3.2.1 ax b a n

Case (i)

i.e. y n n! a n
Case (ii)
If m<n,(i.e. if n>m) which means if we further differentiate the above
expression, the
m
right hand site yields zero. Thus D n ax b 0 if m n

Case (iii)

.
3.

If m>n, then y n mm 1m 2......m n 1ax b a n becomes


mm 1m 2......m n 1m n !
ax bmn a n

m n!
m!
ax bmn a n
i.e
yn
m n!
mn

ax bm
Let

ax b

ax b

Differentiating w.r.t x
m1
m1
y1 max b
a 1max b
a

y 2 1m m 1ax b
a 1 mm 1ax b
3
m3 3
y3 1 mm 1m 2ax b
a
Similarly, we get
m11

m 2

a2

y 4 1 mm 1m 2m 3ax b
a4

n
m n n
y n 1 mm 1m 2.....m n 1ax b
a
This may be rewritten as
n

1 m n 1m n 2.....m 1mm 1!
ax bmn a n
yn
m 1!
4

DEPT OF MATHS, SJBIT

m 4

Page 8

ENGINEERING MATHEMATICS-I
yn

or
4.

15MAT11

1 m n 1! n

a
m 1!ax bmn

1
ax b
Putting m 1, in the result

(1) n (m n 1) ! n
1
D

a
m
m n
(ax b) (m 1) !(ax b)
1 (1) n (1 n 1) ! n
we get D n
a

1 n
(ax b) (1 1) !(ax b)
n

1
(1) n n ! n
or D n

1 n
(ax b) (ax b)
Find the nth derivative of the following examples

(b) log (4 x 3)e 5 x7

1. (a) log( 9 x 2 1)

(c) log 10

(3x 5) 2 (2 3x)
( x 1) 6

Sol: (a) Let y log( 9 x 2 1) log(3x 1)(3x 1)


( log( AB) log A log B )
y log( 3x 1) log( 3x 1)
dn
dn
y n n log( 3x 1) n log( 3x 1)
dx
dx
n 1
(1) (n 1) ! n (1) n 1 (n 1) ! n
i.e y n
(3)
(3)
(3x 1) n
(3x 1) n

(b) Let y log (4 x 3)e 5 x7 log( 4 x 3) log e 5 x7


log( 4 x 3) (5x 7) log e e ( log A B B log A )
( log e e 1 )
y log( 4 x 3) (5x 7)

yn

(1) n 1 (n 1) ! n
(4) 0
(4 x 3) n

D(5x 6) 5
D 2 (5x 6) 0
D n (5x 1) 0 (n 1)

(c) Let y log 10

(3x 5) 2 (2 3x)
( x 1) 6

2
1
(3x 5) (2 3x)

6
log e 10
( x 1)

DEPT OF MATHS, SJBIT

log 10 X

log e X
log e 10

Page 9

ENGINEERING MATHEMATICS-I

15MAT11

(3x 5) 2 (2 3x)
1 1
log

log e 10 2
( x 1) 6

log A B B log A

A
log log A log B
B
1

log( 3x 5) 2 log( 2 3x) log( x 1) 6


2 log e 10
1
2 log( 3x 5) log( 2 3x) 6 log( x 1)
y
2 log e 10
Hence,

yn

(1) n1 (n 1) ! n (1) n1 (n 1) !
(1) n1 (n 1) ! n
1
n
(
3
)

3
)

6
.
(1)
2.
2 log e 10
(3x 5) n
(2 3 x) n
( x 1) n

2. (a) e 2 x 4 6 2 x4

(b) cosh 4 x cosh 2 4 x


1
1
(c) e x sinh 3x cosh 2 x (d)

(6 x 8) 5
4
(4 x 5) (5 x 4)

Sol: (a) Let y e 2 x 4 6 2 x 4


e2x e4 62x 64
y e 4 (e 2 x ) 1296(6 2 x )

dn
dn
hence
y n e 4 n (e 2 x ) 1296 n (6 2 x )
dx
dx
4
n 2x
n
e 2 e 1296 2 (log 6) n 6 2 x
(b) Let y cosh 4 x cosh 2 4 x

hence,

e 4 x e 4 x e 4 x e 4 x


2
2

1 4x
1
e e 4 x (e 4 x ) 2 (e 4 x ) 2 2(e 4 x )(e 4 x )
2
4
1 4x
1
y e e 4 x e 8 x e 8 x 2
2
4
1
1
y n 4 n e 4 x (4) n e 4 x 8 n e 8n (8) n e 8n 0
2
4
x
(c) Let y e sinh 3x cosh 2 x

e 3 x e 3 x e 2 x e 2 x
e x

2
2

x
e
(e 3x e 3x )(e 2 x e 2 x )

4
DEPT OF MATHS, SJBIT

Page 10

ENGINEERING MATHEMATICS-I

15MAT11

ex 5x
e e x e x e 5 x
4
1 4x
e e 2 x 1 e 6 x
4
1
y 1 e 4 x e 2 x e 6 x
4
1
y n 0 (4) n e 4 x (2) n e 2 x (6) n e 6 x
4
1
1
(d) Let y

(6 x 8) 5
4
(4 x 5) (5 x 4)

Hence,

1 dn
dn
1
5
n 6 x 8

n
4
(4 x 5) dx (5 x 4) dx
(1) n n!
(1) n (4 n 1)!
n

(
4
)

(5) n 0
n 1
4 n
(4 x 5)
(4 1)!(5 x 4)

Hence, yn

i.e y n

dn
dx n

(1) n n !
(1) n (3 n) ! n
n
(
4
)

(5)
(4 x 5) n 1
3!(5 x 4) n 4

Evaluate
1. (i)

x2
1
1
(ii)
(iii)
2x 2 7x 6
x 2 6x 8
1 x x2 x3

1
x 2
(iv)
2
x 1 4 x 12 x 9

a (vi) tan

(v) tan 1 x

1 x
x (vii) tan 1

1 x

1
1
. The function can be rewritten as y
( x 4)( x 2)
x 6x 8
This is proper fraction containing two distinct linear factors in the denominator.
So, it can be split into partial fractions as

Sol: (i) Let y

1
A
B
Where the constant A and B are found

( x 4)( x 2) ( x 4) ( x 2)

as given below.

1
A( x 2) B( x 4)

( x 4)( x 2)
( x 4)( x 2)
1 A( x 2) B( x 4) -------------(*)

Putting x = 2 in (*), we get the value of B as B 1

DEPT OF MATHS, SJBIT

2
Page 11

ENGINEERING MATHEMATICS-I

15MAT11

Similarly putting x = 4 in (*), we get the value of A as A 1

1
(1 / 2) (1 / 2)
Hence

( x 4)( x 2) x 4
x2
1 dn 1 1 dn 1
yn

2 dx n x 4 2 dx n x 2

(ii) Let y

1 (1) n n !
1 (1) n n !
n
(
1
)

(1) n

n 1
n 1
2 ( x 4)
2 ( x 2)

1
1
1
(1) n n !

n 1
n 1
2
( x 2)
( x 4)

1
1
1

2
3
2
1 x x x
(1 x) x (1 x) (1 x)(1 x 2 )
1
1
ie y

(1 x)(1 x)(1 x) (1 x) 2 (1 x)

Though y is a proper fraction, it contains a repeated linear factor (1 x) 2


in its
denominator. Hence, we write the function as
A
B
C
in terms of partial fractions. The constants
y

2
(1 x) (1 x)
1 x
A, B, C
are found as follows:
y

1
A
B
C

2
2
1 x
(1 x) (1 x) (1 x) (1 x)

ie 1 A(1 x)(1 x) B(1 x) C (1 x) 2 -------------(**)


Putting x = 1 in (**), we get B as B 1
2
1
Putting x = -1 in (**), we get C as C
4
Putting x = 0 in (**), we get 1 A B C
A 1 B C 1 1 1 1
2
4
4
A 1
4
(1 / 4)
(1 / 2)
(1 / 4)

Hence, y
2
(1 x) (1 x)
(1 x)

DEPT OF MATHS, SJBIT

Page 12

ENGINEERING MATHEMATICS-I

15MAT11

1 (1) n n ! n 1 (1) n (2 n 1) ! n 1 (1) n n ! n


(1)
(1)
(1)
yn
2 n
n 1
4 (1 x) n1
2 (2 1) !(1 x)
4 (1 x)

1 (1) n (n 1) !
1
1
1
(1) n n !

n 1
n 1
4
(
1

x
)
(
1

x
)

2 (1 x)n 2

x2
(iii) Let y 2
(VTU July-05)
2x 7 x 6
This is an improper function. We make it proper fraction by actual division
and later
spilt that into partial fractions.
1
( 7 x 3)
i.e x 2 (2 x 2 7 x 6) 2 2
2 2x 7 x 6

7
1
2 x3
Resolving this proper fraction into partial fractions,
y
2 (2 x 3)( x 2)
we get

1 A
B
. Following the above examples for finding A &

2 (2 x 3) ( x 2)

1 92
(4)

2 2 x 3 x 2

B, we get

Hence, y n 0

(1) n n !

9 (1) n n !
n
(
2
)

4
(1) n


n 1
n 1
2 (2 x 3)
( x 2)

9 (2) n

i.e y n (1) n n ! 2
n 1
n 1
( x 2)
(2 x 3)
( x 2)
x
(iv) Let y
2
( x 1) 4 x 12 x 9
(i)
(ii)
Here (i) is improper & (ii) is proper function. So, by actual division (i)
becomes
x 2
1

1
. Hence, y is given by
x 1
x 1

1
1
y 1
[ (2 x 3) 2 4 x 2 12 x 9 ]

2
x

1
(
2
x

3
)

Resolving the last proper fraction into partial fractions, we get


x
A
B

. Solving we get
2
(2 x 3) (2 x 3) 2
(2 x 3)
DEPT OF MATHS, SJBIT

Page 13

ENGINEERING MATHEMATICS-I
A 1

and B 3

15MAT11

32
1 12

y 1

2
1 x (2 x 3) (2 x 3)
(1) n n ! n 1 (1) n n !
3 (1) n (n 1) ! n
n
(
1
)

(
2
)

(2)
yn 0

n
n 1
(1 x)
2 (2 x 3)
2 (2 x 3)n 2

(v) tan 1 x

Let y tan 1 x
y1

1 x

a
1
2
2
a x a

a
y n D n y D n1 ( y1 ) D n1 2
2
x a
a
a
Consider 2

2
( x ai)( x ai)
x a
A
B
, on resolving into partial fractions.

( x ai) ( x ai)
1
1
2
i
2i , on solving for A & B.

( x ai ) ( x ai )
1
a
1 2i
n 1
2i
D n1 2
D n1
D

2
x a
x ai
x ai
n 1
n 1
1 (1) (n 1) ! 1 (1) (n 1) !

-----------(*)
n
n
2i ( x ai)
2i ( x ai)

We take transformation x r cos


x ai r cos i sin rei

a
a r sin where r x 2 a 2 , tan 1
x

x ai r cos i sin re i

x ai n

now(*) is y n

yn

1
e in
1
e in

,
r n e in
r n x ai n
rn

n 1

1 n 1! in
e

2 i rn

e in

1n1 2 i sin n 1n1 n 1! sin n


2 i rn

DEPT OF MATHS, SJBIT

rn

Page 14

ENGINEERING MATHEMATICS-I
(vi) Let y tan 1 x

15MAT11

.Putting a = 1 in Ex.(v) we get

y n which is same as above with r x 2 1 tan 1 1

cot 1 x or x cot

1
1

sin n
n
r
cos ec n
n 1
D n tan 1 x 1 n 1 ! sin n sin n where cot 1 x

r cot 2 1 cos ec

1 x
(vii) Let y tan 1

1 x
put x tan tan 1 x
1 tan
y tan 1
1 tan
tan 1 tan( 4 )

tan

1 tan

4
1 tan

tan 1 ( x)

tan 1 ( x)

4
y n 0 D n (tan 1 x)
n 1
n 1
1 (1) (n 1) ! 1 (1) (n 1) !

n
n
2i ( x ai)
2i ( x ai)

nth derivative of trigonometric functions:


1. sin(ax b) .
Let y sin(ax b) . Differentiating w.r.t x,

y1 cos(ax b).a As sin( x ) cos x


2
We can write y1 a sin(ax b / 2).
again differentiating w.r.t x, y 2 a cos(ax b / 2).a

Again using sin( x ) cos x ,we get y 2 as


2
y 2 a sin(ax b / 2 / 2).a
y 2 a 2 sin(ax b 2 / 2).
i.e.
Similarly, we get
y3 a 3 sin(ax b 3 / 2).
y 4 a 4 sin(ax b 4 / 2).
y n a n sin(ax b n / 2).

DEPT OF MATHS, SJBIT

Page 15

ENGINEERING MATHEMATICS-I

15MAT11

2. e ax sinbx c .
Let y eax sinbx c .....(1)
Differentiating using product rule ,we get
y1 e ax cosbx c b sinbx c ae ax
ax
i.e.
y1 e a sin bx c b cosbx c . For computation of higher order
derivatives
it is convenient to express the constants a and b in terms of the
constants r and
defined by a r cos & b r sin ,so that
r a 2 b 2 and tan 1 b .thus,
a
y1 can be rewritten as
y1 e ax r cos sinbx c r sin cosbx c
or y1 e ax r{sinbx c cos cosbx c cos }
i.e.
y1 re ax sinbx c ...........(2)
Comparing expressions (1) and (2), we write y 2 as

y 2 r 2 e ax sinbx c 2
y3 r 3 e ax sinbx c 3
Continuing in this way, we get
y 4 r 4 e ax sinbx c 4
y5 r 5 e ax sinbx c 5
.
y n r n e ax sinbx c n

D n e ax sinbx c r n e ax sinbx c n , where


r a 2 b 2 & tan 1 b
a

Solve the following:


1. (i) sin 2 x cos 3 x
(ii) sin 3 cos 3 x
(iv) sin x sin 2 x sin 3x (v) e 3 x cos 2 x

(iii) cos x cos 2 x cos 3x


(vi) e 2 x sin 2 x cos 3 x

The following formulae are useful in solving some of the above problems.
1 cos 2 x
1 cos 2 x
(ii ) cos 2 x
(i) sin 2 x
2
2
(iii) sin 3x 3 sin x 4 sin 3 x

(iv ) cos 3x 4 cos 3 x 3 cos x

(v) 2 sin A cos B sin A B sin A B


(vi) 2 cos A sin B sin A B sin A B
(vii) 2 cos A cos B cos A B cos A B

DEPT OF MATHS, SJBIT

Page 16

ENGINEERING MATHEMATICS-I

15MAT11

(viii) 2 sin A sin B cos A B cos A B

1 cos 2 x 1
Sol: (i) Let y sin 2 x cos 3 x
cos 3x 3 cos x
2

4
1
1 n
n
y n 0 2 cos 2 x n 3 cos 3x n 3 cos x n
2 4
2
2
2

sin 3 2 x 1 sin 6 x 3 sin 2 x


sin 2 x
(ii)Let y = sin 3 x cos 3 x

8
8
4
2
1
3 sin 2 x sin 6 x
32
1 n
n n
n
yn
3.2 sin 2 x
6 sin 6 x

32
2
2

(iii) )Let y = cos 3x cos x cos 2 x


1
1
= cos 4 x cos 2 x cos 2 x cos 4 x cos 2 x cos 2 2 x
2
2
1 1
1 cos 4 x
= cos 6 x cos 2 x

2 2
2
1
cos 2 x 1
cos 6 x
1 cos 4 x
4
4
4
n
n

n
2 n cos 2 x
4 cos 4 x

1 n
n
2
2

y n 6 cos 6 x

4
2
4
4

(iv) )Let y = sin 3x sin x sn2 x


1
sin 2 x sin 4 xsin 2 x
2
1
sin 2 2 x sin 4 x sin 2 x
2
1 1 cos 4 x 1

=
sin 2 x sin 6 x
2
2
2

1 cos 4 x 1


sin 2 x sin 6 x
4
4

yn

1 n
n

4 cos 4 x

4
2

n
2 sin 2 x
2

n
6 sin 6 x
2

(v) Let y e 3 x cos 2 x


y n re3 x cos2 x n

2
where r 32 2 2 13 & tan 1
3
DEPT OF MATHS, SJBIT

Page 17

ENGINEERING MATHEMATICS-I

(vi) Let y = e 2 x sin 2 x cos 3 x

15MAT11

1 cos 2 x 1
cos 3x 3 cos x
2
4
2x
1 cos 2 x e
y = e 2 x sin 2 x cos 3 x e 2 x
4 cos 3x 3 cos x
2

1
1
y e 2 x e 2 x cos 2 x e 2 x cos 3x 3e 2 x cos x
2
4
Hence,
1
1
y n 2 n e 2 x r1n e 2 x cos2 x n1 r2n e 2 x cos3x n 2 3r3n e 2 x cosx n 3
2
4

We know that sin 2 x cos 3 x

where r1 2 2 2 2 8 ; r2 2 2 32 13 ; r3 2 2 12 5
2
2

3
2

1
2

1 tan 1 ; 2 tan 1 ; 3 tan 1 ;

Leibnitzs Theorem
Leibnitzs theorem is useful in the calculation of nth derivatives of product of two
functions.
Statement of the theorem:
If u and v are functions of x, then
n
D (uv) D n uv nC1 D n1uDv nC2 D n2 uD 2 v .... nCr D nr uD r v ...uD n v ,
nn 1
n!
d n
where D
, C1 n , n C 2
,........, n C r
2
r!n r !
dx
Examples
1. If x sin t , y sin pt prove that
1 x 2 y n2 2n 1xy n1 p 2 n 2 yn 0
Solution: Note that the function y f (x) is given in the parametric form with a
parameter t.
So, we consider
dy dy dt p cos pt
(p constant)

dx dx dt
cos t

p 2 cos 2 pt p 2 (1 sin 2 pt ) p 2 (1 y 2 )
dy
or

cos 2 t
1 sin 2 t
1 x2
dx
or 1 x 2 y12 p 2 1 y 2
So that 1 x 2 y12 p 2 1 y 2
Differentiating w.r.t. x,
2
2
2
1 x 2 y1 y 2 y1 2 x p 2 yy1 0
2

DEPT OF MATHS, SJBIT

Page 18

ENGINEERING MATHEMATICS-I

1 x y

15MAT11

--------------- (1)
[ 2y1 , throughout]
xy1 p 2 y 0
Equation (1) has second order derivative y 2 in it. We differentiate (1), n times,
term wise,
using Leibnitzs theorem as follows.
D n 1 x 2 y 2 xy1 p 2 y 0
i.e D n (1 x 2 ) y 2 D n xy1 D n p 2 y 0
---------- (2)
2

(a)
(b)
(c)
Consider the term (a):
D n 1 x 2 y 2 . Taking u y 2 and v (1 x 2 ) and applying Leibnitzs theorem

we get

D n uv D n uv nC1 D n1uDv nC2 D n2 D 2 v nC3 D n3uD 3 v ...

i.e
D n y 2 (1 x 2 ) D n ( y 2 ).(1 x 2 ) nC1 D n1 ( y 2 ).D(1 x 2 ) nC2 D n2 ( y 2 ) D 2 (1 x 2 ) nC3 D n3 ( y2 ) D 3 (1 x 2 )

y ( n ) 2 x 2 ) ny ( n1) 2 .(2 x)

n(n 1)
n(n 1)(n 2)
y( n2) 2 .(2)
. y ( n3) 2 .(0) ...
2!
3!

----------- (3)
D n 1 x 2 y 2 1 x 2 y n2 2nxy n1 n(n 1) y n
Consider the term (b):
D n xy1 . Taking u y1 and v x and applying Leibnitzs theorem,
we get
D n y1 ( x) D n ( y1 ).( x) nC1 D n1 y1 .D( x) nC2 D n2 ( y1 ).D 2 ( x) ....
n(n 1)
y ( n )1 .x ny ( n1)1
y ( n2) 2 (0) ....
2!
D n xy1 xy n1 ny n

---------- (4)

Consider the term (c):


--------- (5)
D n ( p 2 y) p 2 D n ( y) p 2 y n
Substituting these values (3), (4) and (5) in Eq (2) we get
1 x 2 yn2 2nxy n1 n(n 1) yn xy n1 nyn p 2 yn 0
ie 1 x 2 y n2 (2n 1) xy n1 n 2 y n ny n ny n p 2 y n 0

1 x 2 y n2 (2n 1) xy n1 p 2 n 2 y n 0 as desired.

2. If sin 1 y 2 log( x 1) or y sin2 log( x 1) or y sin log( x 1) 2 or

y sin log( x 2 x 1) , show that x 1 y n 2 2n 1x 1y n1 n 2 4 y n 0


(VTU Jan-03)
Sol: Out of the above four versions, we consider the function as
sin 1 ( y) 2 log( x 1)
Differentiating w.r.t x, we get
2

DEPT OF MATHS, SJBIT

Page 19

ENGINEERING MATHEMATICS-I

15MAT11

2
2
( y1 )
ie ( x 1) y1 2 1 y
2
x

1 y
Squaring on both sides
x 12 y12 4(1 y 2 )
Again differentiating w.r.t x,
x 12 2 y1 y2 y12 2( x 1) 4(2 yy1 )
1

or
or

x 12 y2 ( x 1) y1 4 y (2 y1 )
-----------*
x 12 y2 ( x 1) y1 4 y 0

Differentiating * w.r.t x, n-times, using Leibnitzs theorem,


n

n(n 1) n2
2
n 1
D ( y 2 )(2) D n ( g1 )( x 1) nD n1 y1 (1) 4 D n y 0
D y 2 ( x 1) nD ( y 2 )2( x 1)
2!

On simplification, we get
x 12 yn2 2n 1x 1yn1 n 2 4 yn 0

3. If x tan(log y) , then find the value of


1 x 2 yn1 2nx 1yn n(n 1) yn1
Sol: Consider x tan(log y)

(VTU July-04)

i.e. tan 1 x log y


or y e tan x
Differentiating w.r.t x,
1
1
y
y1 e tan x .

2
1 x
1 x2
ie 1 x 2 y1 y 0 -----------*
1 x 2 y1 y
We differentiate * n-times using Leibnitzs theorem,

We get

D n 1 x 2 y1 D n ( y) 0

ie.
D ( y1 )(1 x 2 ) nC1 D n1 ( y1 ) D(1 x 2 ) nC2 D n2 ( y1 ) D 2 (1 x 2 ) .... D n y 0

n(n 1)
y n1 (2) 0 .... y n 0
ie y n1 (1 x 2 ) ny n (2 x)
2!

2
1 x yn1 2nx 1yn n(n 1) yn1 0

4. If y m y m 2 x , or y x x 2 1 or y x x 2 1
Show that x 2 1y n2 (2n 1) xy n1 n 2 m 2 y n 0
1

Sol: Consider

y
1

2x

DEPT OF MATHS, SJBIT

(VTU Feb-02)

1
2x
1
ym

1 0 Which is quadratic equation in y

2x y

Page 20

ENGINEERING MATHEMATICS-I
y

(2 x) (2 x) 2 4(1)(1) 2 x 4 x 2 4

2(1)
2

1
2x 2 x 2 1
x x2 1 y m x x2 1
2

y x x2 1

Let us take y x x 2 1

or

y x x2 1

y1 m x x 2 1

so, we can consider y x x 2 1

15MAT11

1
1

(
2
x
)

2
2 x 1

m 1
x 2 1 x
y1 m x x 2 1

x 2 1

or

m 1

x 1y my . On squaring
2

1 y12 m 2 y 2 .

Again differentiating w.r.t x,

or
or

1 2 y1 y 2 y12 (2 x) m 2 (2 yy1 )

1 y 2 xy1 m 2 y (2 y1 )

1 y 2 xy1 m 2 y 0

------------(*)

Differentiating (*) n- times using Leibnitzs theorem and simplifying, we get


x 2 1yn2 (2n 1) xy n1 n 2 m2 yn 0

POLAR CURVES
Angle between Polar Curves:
Introduction:- We are familiar with Cartesian coordinate system for specifying a point
in the xy plane. Another useful system for similar purpose is Polar coordinate system,
and the curves specified by these coordinates are referred to as polar curves.
A polar curve by name three-leaved rose is displayed below:

DEPT OF MATHS, SJBIT

Page 21

ENGINEERING MATHEMATICS-I
=

3
4

=0

15MAT11

3
2

Any point P can be located on a plane with co-ordinates r ,


called polar co-ordinates of P where r = radius vector OP,(with pole O)
= projection of OP on the
initial axis OA.(See Fig.)
The equation r f is known as a polar curve.
Polar coordinates r , can be related with Cartesian coordinates x, y through
the relations
Fig.1. Polar coordinate system
x r cos & y r sin .

Theorem 1: Angle between the radius vector and the tangent:


T

i.e., With usual notation prove that tan r

d
dr

Proof:- Let be the angle between the radius vector OPL


r
and the tangent TPT 1 at the point `P` on the polar

O
curve r f . (See fig.2)
r = f()
From Fig.2,
Fig.2. Angle between radius
vector and the tangent

tan tan
tan tan
1 tan tan
dy tan tan
i.e.

................. (1)
dx 1 tan tan

DEPT OF MATHS, SJBIT

Page 22

P(r, )

1
1
1

ENGINEERING MATHEMATICS-I

On the other hand, we have x r cos ; y r sin differentiating these,


w.r.t ,
dy
dx
dr
dr
r sin cos
r cos sin

&
d
d
d
d
dr
dy
r cos sin

dy
dr
d

dividing the Nr & Dr by


cos
dx dx
d
dr
d r sin cos

d
r d
tan
dy
dr

dx rd dr tan 1
d
dy tan r dr
i.e.
.(2)

dx 1 tan rd
dr
Comparing equations (1) and (2)
we get tan r d
dr
1
dr

Note that cot

r d
A Note on Angle of intersection of two polar curves:If 1 and 2 are the angles between the common radius vector and the tangents
at the point of intersection of two curves r f1 and r f 2 then the

15MAT11

angle intersection of the curves is given by 1 2


Theorem 2: The length p of perpendicular from pole to the tangent in a polar curve
1
1
1 dr 2
i.e.(i) p r sin
or (ii)
2 4

2
p
r
r d
Proof:- In the Fig.3, note that ON = p, the length of the perpendicular from the pole to
the tangent at p on r f .from the right angled triangle OPN,
ON
sin
ON OPsin
OP
i.e. p r sin ..............(i)
1
1
1
Consider
cos ec
p r sin r
1
1
1
2 2 cos ec 2 2 1 cot 2
p
r
r
2
1
1 1 dr


1
p 2 r 2 r d

P(r, )

r = f ()

P
N

Fig.3 Length of the perpendicular


from the pole to the tangent

DEPT OF MATHS, SJBIT

Page 23

ENGINEERING MATHEMATICS-I

1
1
1
2 4
2
p
r
r

15MAT11

dr

............(ii )
d

1
1
du
Note:-If u , we get 2 u 2

r
p
d

In this session, we solve few problems on angle of intersection of polar curves and pedal
equations.
Examples:Find the acute angle between the following polar curves
1.
(VTU-July-2003)
r a1 cos and
r b1 cos
2

r sin cos and r 2 sin

3.

r 16 sec 2

4.

r a log and r a

(VTU-July-2004)

2 and r 25 cos ec 2

5. r

(VTU-July-2005)

log

a
a
and r
1
1 2

Sol:
1. Consider
r a1 cos
Diff w.r.t

Consider
r b1 cos
Diff w.r.t

dr
a sin
d
d a 1 cos
r

dr
a sin
2 cos 2
2
tan 1

2 sin
cos
2
2

dr
b sin
d
d b1 cos
r

dr
b sin
2 sin 2
2
tan 1

2 sin
cos
2
2

cot

i.e tan 1 tan

1
2
2
2
2
Angle between the curves

tan

2
tan 1 tan

1 2

1 2 2 2 2 2
Hence, the given curves intersect orthogonally.
2. Consider
r sin cos
Diff w.r.t
DEPT OF MATHS, SJBIT

Consider
r 2 sin
Diff w.r.t
Page 24

ENGINEERING MATHEMATICS-I

15MAT11

dr
cos sin
d
d sin cos
r

dr cos sin
tan 1
( Nr & Dr cos )
tan 1
1 tan
tan 1
i.e tan 1
tan
4
1 tan

dr
2 cos
d
d 2 sin
r

dr 2 cos

tan 2 tan

Angle between the curves = 1 2

3.

Consider

Diff w.r.t
dr
50 cos ec 2 cot . 1
2
2 2
d
25 cos ec 2 cot
2
2



25 cos ec
d
2
r

dr 25 cos ec cot
2
2
tan tan tan
2
2

Angle of intersection of the curves = 1 2

4.

Consider
r a log
Diff w.r.t
dr a

d
d
r
a log
a
dr

tan 1 log ..........(i)


We know that
DEPT OF MATHS, SJBIT

r 25 cos ec 2


2 2
16 sec
d
2
r

dr 16 sec tan
2
2
tan cot tan
2
2
2
1

Consider

r 16 sec
2
Diff w.r.t
dr
32 sec 2 tan . 1
2
2 2
d
16 sec tan
2

2
Consider
r a
log
Diff w.r.t
dr
2
a log . 1

d
log 2
d

log
dr
a

tan 2 log ..........(ii )

Page 25

ENGINEERING MATHEMATICS-I
tan1 2

15MAT11

tan 1 tan 2
1 tan 1 tan 2

log log
1 log log
2 log
............(iii )
i.e tan1 2
2
1 log

From the data: a log r a

log 1 or log 1

log
As is acute, we take by =1 e
Substituting e in (iii), we get
2e log e
2e
tan1 2

2
2
1 e log e
1 e
2e
1 2 tan 1
2
1 e

NOTE

log

5. Consider

a
as
r
1
1 1 1 1

1
r
a
a
Diff w.r.t
1 dr 1 1
2

2
r d a
1 dr
r

r d a 2
d a 2
r

dr
r

tan 1

a 2

tan1 1

e
e

Consider
a
r
1 2

1 2 a

Diff w.r.t
dr
r d
2r 1 dr

a
r d
d a
i.e r

dr 2r
2 a

tan 2

a 1 2

2 a

tan 2

1 2
2

Now, we have
a
a
r
a 1 2 a 1
2
1
1
or 3 1 3 1 or 1
tan1 2 & tan 2 1
DEPT OF MATHS, SJBIT

Page 26

ENGINEERING MATHEMATICS-I
Consider tan 1 2

15MAT11

tan 1 tan 2
1 tan 1 tan 2

2 1
3 3
1 2 1

1 2 tan 1 3

Pedal equations (p-r equations):- Any equation containing only p & r is


known as pedal equation of a polar curve.
Working rules to find pedal equations:(i) Eliminate r and from the Eqs.: (i) r f & p r sin
2

1
1
1 dr
(ii) Eliminate only from the Eqs.: (i) r f & 2 2 4

p
r
r d
Find the pedal equations for the polar curves:2a
1.
1 cos
r
2. r e c cot
3. r m a m sin m b m cos m
(VTU-Jan-2005)
l
1 e cos
4.
r
Sol:
2a
1. Consider
1 cos .(i)
r
Diff. w.r.t
dr
2a 1 2
sin
r d
1 dr r sin

r d
2a
d
2a 1
r

dr
r sin
2 sin 2

1 cos
2
tan

tan
2

sin
2 sin
cos
2
2
tan tan
2
2
Using the value of is p r sin , we get
p r sin r sin .............(ii )
2
2
Eliminating between (i) and (ii)

DEPT OF MATHS, SJBIT

Page 27

ENGINEERING MATHEMATICS-I
p 2 r 2 sin 2

15MAT11

2
1 cos r 2a
r2


2
2

2 r

[See eg: - (i)]

p 2 ar.

This eqn. is only in terms of p and r and hence it is the pedal equation of the polar
curve.
2. Consider r e cot
Diff. w.r.t
dr
e cot cot r cot
d
We use the equation

r e

cot

1
1 1 dr
2 4
2
p
r
r d
1 1
2
2 4 r cot
r
r
1 1
1
1
2 4 cot 2 2 1 cot 2 2 cos ec 2
r
r
r
r
1
1
2 cos ec 2
2
p
r

p2 r

cos ec 2

or r 2 p 2 cos ec 2 is the required pedal equation

3.Consider r a sin m b cos m


Diff. w.r.t
dr
mrm 1
a m m cos m bm ( m sin m )
d
m
r dr
a m cos m b m sin m
r d
1 dr
a m cos m b m sin m
m
r d
a sin m b m cos m
m

a m cos m b m sin m
a m sin m b m cos m
1 1
Consider p r sin , cos ec
p r
1
1
2 cos ec 2
2
p
r
1
2 1 cot 2
r
cot

DEPT OF MATHS, SJBIT

Page 28

ENGINEERING MATHEMATICS-I

15MAT11

2
1 a m cos m bm sin m

2 1 m
r a sin m bm cos m

2
2
1 a m sin m bm cos m a m cos m bm sin m

2
2
m
m
r

a sin m b cos m

1
1 a 2m b2m

p2 r 2 r 2m
r 2 m 1
2
is the required p-r equation
p 2m
a b2 m
4. Consider l

1 cos

Diff w.r.t
dr
1 dr

l 2
e sin l r 1 r
e sin
r d
d
l cot e sin
r
cot r e sin
l
1
1
We have 2 2 1 cot 2 (see eg: 3 above)
p
r

Now

1
1 l 2 e2 r 2 sin 2

p 2 r 2
l2

2 2
1
1 e r 2 sin 2
2
l
r

1 e cos l
r

e cos

lr
r

l r
l r
2
2
cos

sin 1 cos 1
re
re

2
l r 2
2 2
l e r 1

re
1
1

p2 r2
l2

On simplification

1 e2 1 2

p 2 e 2 lr

DEPT OF MATHS, SJBIT

Page 29

ENGINEERING MATHEMATICS-I

15MAT11

DERIVATIVES OF ARC LENGTH:


Consider a curve C in the XY plane. Let A be a fixed point on it. Let P and Q be
two neighboring positions of a variable point on the curve C. If s is the distance of P
from A measured along the curve then s is called the arc length of P. Let the tangent to
C at P make an angle with X-axis. Then (s,) are called the intrinsic co-ordinates of
the point P. Let the arc length AQ be s + s. Then the distance between P and Q
measured along the curve C is s. If the actual distance between P and Q is C. Then
s=C in the limit Q P along C.

y
Q
s
s

P(s, )

0
i.e.

s
1
QP C
Lt

Cartesian Form:

Q( x x, y y)
C s

P( x, y)

DEPT OF MATHS, SJBIT

Page 30

ENGINEERING MATHEMATICS-I
Let y f ( x)

be

the

Cartesian

15MAT11

equation

of

the

curve

and

let

P( x, y) and Q( x x, y y) be any two neighboring points on it as in fig.

Let the arc length PQ s and the chord length PQ C . Using distance between two
points formula we have PQ2 = ( C)2 =( x)2 +( y)2

C
y

1
x
x
2

C
y
or
1
x
x

s s C s
y

x C x C
x

We note that x 0 as Q P along C, also that when Q P,

s
1
C

When Q P i.e. when x 0, from (1) we get

ds
dy
1
dx
dx

(1)

Similarly we may also write


x
s s C s

1
y C y C
y

and hence when Q P this leads to

dx
ds
1
dy
dy

(2)

Parametric Form: Suppose x x(t ) and y y(t ) is the parametric form of the curve C.
Then from (1)
2

dy
2
2

ds
1
dx dy
1 dt


dx
dx
dt dt
dx dt
dt

ds ds dx
dx dy

dt dx dt
dt dt

(3)

Note: Since is the angle between the tangent at P and the X-axis,
DEPT OF MATHS, SJBIT

Page 31

ENGINEERING MATHEMATICS-I
we have

15MAT11

dy
tan
dx

ds
2
1 y 1 tan 2 sec
dx

Similarly

ds
1
1
1
1
1 cot 2 co sec
2
2
dy
tan
y
i.e. cos
2

dx
dy
and sin
ds
ds

dx dy
2
2
2
1 ds dx dy
ds ds
We can use the following figure to observe the above geometrical connections
among dx, dy, ds and .

dy
ds

dx

DEPT OF MATHS, SJBIT

Page 32

ENGINEERING MATHEMATICS-I

15MAT11

Polar Curves:
Suppose r f ( ) is the polar equation of the curve C and P(r, ) and Q(r r, )
be two neighboring points on it as in figure:

Q(r r, )
N
s
P(r,)

Consider PN

OQ.

In the right-angled triangle OPN, We have sin

PN PN

PN r sin r
OP
r

since Sin = when is very small.

From the figure we see that, cos

ON ON

ON r cos r (1) r
OP
r
cos 1 when 0

NQ OQ ON (r r ) r r

From

PNQ, PQ

PN 2 NQ2 i.e, ( C )2 (r )2 ( r )2

C
S S C S 2 r
r
r2

C C


2

We note that when Q P along the curve, 0 also

S
1
C

dS
dr
when Q P,
r2
(4)
d
d
Similarly, ( C )2 (r ) 2 ( r ) 2

DEPT OF MATHS, SJBIT

C

1 r2
r
r

Page 33

ENGINEERING MATHEMATICS-I
S S C S


1 r2
r C r C
r

and

15MAT11

dS
d
when Q P, we get
1 r 2
(5)
dr
dr
2

Note:

We know that tan r

d
dr

ds
dr
2
2
2
2
r2
r r cot r 1 cot rco sec
d
d

Similarly
ds
d
2
1 r2
1 tan sec
dr
dr
2

dr
cos
ds

and

d 1
sin
ds r

The following figure shows the geometrical connections among ds, dr, d and

ds

r d

dr
Thus we have :
2

ds
dy
1 ,
dx
dx

dx
ds
ds
dx dy
1 ,

dy
dt
dt dt
dy

DEPT OF MATHS, SJBIT

Page 34

ENGINEERING MATHEMATICS-I
ds
d
1 r2

dr
dr

Example 1:

and

ds
dr
r2

d
d

15MAT11
2

ds
ds
and
for the curve x 2/3 y 2/3 a 2/3
dx
dy
-1

x 2/3 y 2/3

x3
2
2
y 3
a 2/3 x -1/3 y -1/3 y ' 0 y ' -1
3
3
x
y3
2

ds
y 2/3
dy
1 1 2/3
dx
x
dx

Hence

x 2/3 y 2/3
a 2/3 a


x 2/3
x 2/3 x

1/3

Similarly
2

dx
ds
x 2/3
1 1 2/3
dy
y
dy

x 2/3 y 2/3
y 2/3

1/3

a 2/3 a

y 2/3 y

Example 2: Find

a2
ds
for the curve y a log 2 2
dx
a -x

y a log a 2 a log a 2 x 2

dy
2ax
2 x
a 2
2
2
2
dx
a x a x

ds
4a 2 x 2
dy
1 1 2
dx
dx
a x2

x 2 4a 2 x 2

x2

a
a

2
2

x2
x2

2
2

a2 x2
a2 x2

Example 3: If x aet sint, y aet cost, find ds


dt

x ae t sint

dx
ae t sint ae t cost
dt

DEPT OF MATHS, SJBIT

Page 35

ENGINEERING MATHEMATICS-I
y aet cost

15MAT11

dy
aet cost aet sint
dt

ds
2
2
dx dy
a 2e2t cos t sin t a 2e2t cos t sin t
dt
dt dt

aet 2 cos 2t sin2t a 2 et

a b a b 2 a 2 b2
2

t
ds

Example 4: If x a cos t log tan , y a sin t, find


2
dt

sec 2 t

dx
1
2 a sin t
a sin t

t
t
dt

2 tan t

2
sin
cos

2
2
1 sin2t a cos 2t

1
a sin t
a

a cost cot t
sin t
sin t
sin t

dy
a cos t
dt
2

ds
dx dy

dt
dt dt

a 2 cos 2t cot 2t a 2cos 2t

a 2cos 2t cot 2t 1

a 2cos 2t co sec2 t a 2cot 2t


a cot t

Example 5: If x a cos3 t, y sin 3 t, find

ds
dt

dx
dy
3 a cos 2t sin t ,
3 a sin 2t cos t
dt
dt
2

ds
dx dy

dt
dt dt

9a 2cos 4t sin2t 9a 2 sin4t cos 2t

9a 2cos 2t sin2t cos 2t sin2t 3 a sin t cos t

DEPT OF MATHS, SJBIT

Page 36

ENGINEERING MATHEMATICS-I

15MAT11

ds
is constant
d
dr
dr a 2
r 2 a 2cos 2 2r
2a 2 sin2

sin2
d
d
r

Example 6: If r 2 a 2 cos 2 , Show that r

ds
a4
1 4
dr
2
r2

sin2 2
r a 4 sin 2 2

2
d
r
r
d

ds
r 4 a 4 sin2 2 a 4cos 2 2 a 4 sin2 2
d

a 2 cos 2 sin2 2 a 2 constant

ds
is constant for r 2 a 2cos2
d

k2 r2
ds
r
Example 7: For the curve cos-1
, Show that r is constant.
r
dr
k

2r
2
2
r
k r (1)
2
2
r2 k 2 r2
1
d
1
1
2 k r


dr
r2
k2 r2
r2 k2 r2
r2 k
1 2
k

1
k2 r2

k2
r2 k2 r2

ds
d
1 r2
dr
dr

1 r

Hence r

r2
r2

r 2 k 2
r2 k2 r2

k2 r2
r

r2 k2 r2 k

r
r

ds
k (constant)
dr

Example 8: For a polar curve r f show that


We know that cos

ds
r
ds r 2

dr
p
r 2 p 2 d

dr
d 1
and
sin
ds
ds r

r 2 p2
dr
p2
cos 1 sin2 1 2
ds
r
r

DEPT OF MATHS, SJBIT

p rsin

Page 37

ENGINEERING MATHEMATICS-I

15MAT11

ds
r

2
dr
r p2

Also

ds
r
r
r2

d sin p
p
r

CURVATURE:
Consider a curve C in XY-plane and let P, Q be any two neighboring points on it.
Let arc AP=s and arc PQ=s. Let the tangents drawn to the curve at P, Q respectively
make angles and + with X-axis i.e., the angle between the tangents at P and Q is
. While moving from P to Q through a distances, the tangent has turned through the
angle . This is called the bending of the arc PQ. Geometrically, a change in
represents the bending of the curve C and the ratio

represents the ratio of bending of


s

C between the point P & Q and the arc length between them.

y
Q
s
P
C

+
x

Rate of bending of Curve at P is

Lt
ds Q P s

This rate of bending is called the curvature of the curve C at the point P and is denoted by
d
(kappa). Thus
We note that the curvature of a straight line is zero since there
ds
exist no bending i.e. =0, and that the curvature of a circle is a constant and it is not equal
to zero since a circle bends uniformly at every point on it
DEPT OF MATHS, SJBIT

Page 38

ENGINEERING MATHEMATICS-I
If 0, then

is called the radius of curvature and is denoted by (rho - Greek letter).

ds
d
Radius of curvature in Cartesian form :

15MAT11

Suppose y = f(x) is the Cartesian equation of the curve considered in figure.

y
c

we have

dy
d2 y
d
d ds
tan y 2 sec 2
1 tan2

dx
dx
dx
ds dx

ds
dy
But we know that
1
dx
dx

dy 2
1
2
2
d 2 y dy d
ds dx
dy
2 1
1

d2y
dx
d
dx
dx ds
dx 2
2

ds 1 y

d
y

This is the expression for radius of curvature in Cartesian form.

DEPT OF MATHS, SJBIT

Page 39

ENGINEERING MATHEMATICS-I

15MAT11

dx 2
1
dy
NOTE: We note that when y=, we find using the formula
d 2x
2
dy

Example 9: Find the radius of curvature of the curve x3+y3 = 2a3 at the point (a, a).
x3 y3 2a3 3x2 3 y 2 y 0 y

x2
hence at a, a , y 1
y2

y 2 2 x x 2 2 y y
2a 3 2a 3
4

y
,
hence
at
a
,
a
,
y

4
4
y
a
a

1 y 2

1 12

4
a

i.e.,

Example 10: Find the radius of curvature for

a
2 2
4

a
2

x y a at the point where it meets

the line y=x.

On the line y x,

x x a i.e 2 x a

or

a
4

a a
i.e., We need to find at ,
4 4

x y a

1
2 x

1
2 y

y 0 i.e y

y
a a
, hence at , , y 1
x
4 4

1
1

x 2 y y y 2 x

Also, y
x

a 1
a 1
(1)

4
a
1 1
42 a
2
( )

a a
4
4 2 2 (1) 4
at , , y

a
a
a
a
4 4

4
4
4

DEPT OF MATHS, SJBIT

Page 40

ENGINEERING MATHEMATICS-I
1 y 2

1 12

4
a

15MAT11

a
a
2 2
4
2

Example 11: Show that the radius of curvature for the curve y 4 Sin x - Sin 2x

at x

is 5 5

y 4 sin x - sin 2x y 4 cos x 2 cos 2x

when x

, y 4cos 2cos 0 2(1) 2


2

Also, y -4 sin x 4 sin 2x and when x


1 y 2

1 22

, y -4 sin 4 sin 4
2

5 5
4

Example 12: Find the radius of curvature for xy2 = a 3 - x3 at (a, 0).
xy 2 a3 x3 y 2 2 xy y 3x 2

3x 2 y 2
and at (a, 0), y
2 xy

In such cases we write

dx
2 xy
dx

and at (a, 0),


0
2
2
dy 3x y
dy

dx

2 dx
y 2 x 2 xy 6 x 2 y
3x y 2
dx
2 xy
d x
dy

dy

Also
2
2
2
2
2
2

dy 3x y
dy
3x y

d 2 x 3a 0 0 2a 0 6a3 2
At a, 0 ,

2
4
2

dy 2
9
a
3a
3
a

dx 2
1
dy
2
d x 2
dy

1 o 2

2
3a

DEPT OF MATHS, SJBIT

or

3a
2

Page 41

ENGINEERING MATHEMATICS-I

15MAT11

1. Find the radius of curvature of the curve


x a log(sect tant), y asect

x a log(sect tant)
dx
a
a sec t (seet tant)

sec t tan t sec 2 t


dt sec t tan t
(seet tant)
dx
a sec t
dt
Also y a sec t gives
dy
a sec t tan t
dt
dy dy dt a sec t tan t
Now , y1

dx dt dx
a sec t
y1 tan t
Differentiating w.r.t x we get
dt
dx

y2 sec 2 t
y2

sec t
a

1 y
we have
2
1

y2

a 1 tan 2 t

sec t
a sec 2 t

DEPT OF MATHS, SJBIT

Page 42

ENGINEERING MATHEMATICS-I

15MAT11

2.

DEPT OF MATHS, SJBIT

Page 43

ENGINEERING MATHEMATICS-I

DEPT OF MATHS, SJBIT

15MAT11

Page 44

ENGINEERING MATHEMATICS-I

DEPT OF MATHS, SJBIT

15MAT11

Page 45

ENGINEERING MATHEMATICS-I

15MAT11

1.

DEPT OF MATHS, SJBIT

Page 46

ENGINEERING MATHEMATICS-I

15MAT11

2.

3.

DEPT OF MATHS, SJBIT

Page 47

ENGINEERING MATHEMATICS-I

DEPT OF MATHS, SJBIT

15MAT11

Page 48

ENGINEERING MATHEMATICS-I

15MAT11

MODULE II
DIFFERENTIAL CALCULUS-II
CONTENTS:
Taylors and Maclaurins theorems for function of o ne
variable50
Indeterminate forms 53
LHospitals rule (without proof)55
Partial derivatives68
Total derivative and chain rule72
Jacobians Direct evaluation.75

DEPT OF MATHS, SJBIT

Page 49

ENGINEERING MATHEMATICS-I

15MAT11

Taylors Mean Value Theorem:


(Generalized Mean Value Theorem):
(English Mathematician Brook Taylor 1685-1731)
Statement:

Suppose a function f (x) satisfies the following two conditions:


(i) f (x) and its first (n-1) derivatives are continuous in a closed interval

a, b

(ii) f (n 1) ( x) is differentiable in the open interval a, b


Then there exists at least one point c in the open interval a, b such that

f (b) f (a) (b a) f (a)

(b a)2
(b a)3
f (a)
f (a) ..
2
3

(b a)n 1 (n 1)
(b a)n ( n)
f
(a)
f (c) (1)
n 1
n
Taking b a h and for 0 1 , the above expression (1) can be rewritten as
.....

h2
h3
hn 1 (n 1)
h n ( n)
f (a)
f (a) ....
f
(a)
f (a h) (2)
2
3
n 1
n
Taking b=x in (1) we may write
f (a h) f (a) hf (a)

f ( x) f (a) ( x a) f (a)

( x a)2
( x a)3
( x a)n 1 (n 1)
f (a)
f (a) ...
f
(a) Rn (3)
2
3
n 1

( x a )n ( n)
Where Rn
f (c) Re mainder term after n terms
n
When n , we can show that Rn 0 , thus we can write the Taylors series as
f ( x) f (a) ( x a) f (a)

( x a)2
( x a )n 1 ( n 1)
f (a ) ...
f
(a) ....
2
n 1

( x a)n ( n)
f (a) (4)
n
n 1

f (a)

Using (4) we can write a Taylors series expansion for the given function f(x) in powers
of (x-a) or about the point a.

DEPT OF MATHS, SJBIT

Page 50

ENGINEERING MATHEMATICS-I

15MAT11

Maclaurins series:
(Scottish Mathematician Colin Maclaurins 1698-1746)
When a=0, expression (4) reduces to a Maclaurins expansion given by
f ( x) f (0) xf (0)

x2
x n 1 ( n 1)
f (0) ...
f
(0) ....
2
n 1

x n ( n)
f (0) (5)
n
n 1

f (0)

Example 1: Obtain a Taylors expansion for f ( x) sin x in the ascending powers of

x up to the fourth degree term.


4

The Taylors expansion for f (x) about

is
4

( x )2
( x )3
( x )4

4 f ( )
4 f ( )
4 f (4) ( ) .... (1)
f ( x) f ( ) ( x ) f ( )
4
4
4
2
4
3
4
4
4

1

f ( x) cos x f cos
4
2
4

1

;
f ( x) sin x f sin
4
2
4

1

f ( x) sin x f sin
4
2
4

1

f ( x) cos x f cos
4
2
4

1

f (4) ( x) sin x f (4) sin
4
2
4
Substituting these in (1) we obtain the required Taylors series in the form

( x )2
( x )3
( x )4
1
1
1
1
1
4
4
4
f ( x)

( x )( )
4
2

) ....

( x )3 ( x ) 4

1
(x 4 )
4
4
f ( x)

...
1 ( x )
4
2
3
4
2

Example 2.: Obtain a Taylors expansion for f ( x) loge x up


to the term containing x 1 and hence find loge(1.1).
4

DEPT OF MATHS, SJBIT

Page 51

ENGINEERING MATHEMATICS-I

15MAT11

The Taylors series for f (x) about the point 1 is


f ( x) f (1) ( x 1) f (1)

( x 1)2
( x 1)3
( x 1) 4 (4)
f (1)
f (1)
f (1) .... (1)
2
3
4

f ( x)

Here f ( x) loge x f (1) log 1 0 ;

f ( x)

1
x

f (4) ( x)

f (1) 1 ;
6
4

f ( x)

2
x3

1
f (1) 1
x

f (1) 2

f (4) (1) 6 etc.,

x
Using all these values in (1) we get
f ( x) log e x 0 ( x 1)(1)

loge x ( x 1)

( x 1)2
( x 1)3
( x 1)4
(1)
(2)
( 6) ....
2
3
4

( x 1)2 ( x 1)3 ( x 1)4

....
2
3
4

Taking x=1.1 in the above expansion we get


log e (1.1) (0.1)

(0.1)2 (0.1)3 (0.1) 4

.... 0.0953
2
3
4

Example 18: Using Taylors theorem Show that


loge (1 x) x

x 2 x3

for 0 1, x 0
2
3

Taking n=3 in the statement of Taylors theorem, we can write


x2
x3
f (a x) f (a) xf (a)
f (a)
f (a x) (1)
2
3
1
1
2
Consider f ( x) loge x f ( x) ; f ( x)
and f ( x)
2
x
x
x3
Using these in (1), we can write,
2
3

2
1 x 1 x
log(a x) log a x
(2)
a 2 a 2 3 (a x)3
For a=1 in (2) we write,
x 2 x3
1
x 2 x3
1
log(1 x) log1 x

3
2
3 (1 x)
2
3 (1 x)3

DEPT OF MATHS, SJBIT

Page 52

ENGINEERING MATHEMATICS-I
Since x 0 and 0, (1 x)3 1 and therefore

15MAT11
1
(1 x)3

x 2 x3
log(1 x) x

2
3

Example 19: Obtain a Maclaurins series for f ( x) sin x up to the term containing x5 .
The Maclaurins series for f(x) is
f ( x) f (0) x f (0)

x2
x3
x 4 (4)
x5 (5)
f (0)
f (0)
f (0)
f (0) .... (1)
2
3
4
5

Here f ( x) sin x f (0) sin 0 0

f ( x) cos x f (0) cos 0 1

f ( x) sin x f (0) sin 0 0

f ( x) cos x f (0) cos 0 1

f (4) ( x) sin x f (4) (0) sin 0 0

f (5) ( x) cos x f (5) (0) cos 0 1

Substituting these values in (1), we get the Maclaurins series for f ( x) sin x as
f ( x) sin x 0 x (1)
sin x x

x2
x3
x4
x5
(0) (1)
(0) (1) ....
2
3
4
5

x3 x5

....
3
5

Indeterminate Forms:
While evaluating certain limits, we come across expressions of the form
0
, , 0 , , 00 , 0 and 1 which do not represent any value. Such expressions are
0
called Indeterminate Forms.
We can evaluate such limits that lead to indeterminate forms by using LHospitals Rule
(French Mathematician 1661-1704).

LHospitals Rule:
If f(x) and g(x) are two functions such that
(i) lim f ( x) 0 and lim g ( x) 0
x a

x a

DEPT OF MATHS, SJBIT

Page 53

ENGINEERING MATHEMATICS-I

15MAT11

(ii) f ( x) andg ( x) exist and g (a) 0

f ( x)
f ( x)
lim
x a g ( x)
x a g ( x )
The above rule can be extended, i.e, if
f ( x)
f ( x)
f ( x)
f (a) 0 and g (a) 0 then lim
lim
lim
.....
x a g ( x)
x a g ( x )
x a g ( x)
Then lim

Note:
0
forms. Here we
,
0
f ( x)
differentiate the numerator and denominator separately to write
and apply the
g ( x)
0

limit to see whether it is a finite value. If it is still in or


form we continue to
0

f ( x)
differentiate the numerator and denominator and write further
and apply the
g ( x)
limit to see whether it is a finite value. We can continue the above procedure till we
get a definite value of the limit.

1. We apply LHospitals Rule only to evaluate the limits that in

2. To evaluate the indeterminate forms of the form 0 , , we rewrite the functions


0

involved or take L.C.M. to arrange the expression in either or and then apply
0

LHospitals Rule.
3. To evaluate the limits of the form 00 , 0 and 1 i.e, where function to the power of
function exists, call such an expression as some constant, then take logarithm on both
0

sides and rewrite the expressions to get or


form and then apply the LHospitals
0

Rule.
4. We can use the values of the standard limits like
sin x
tan x
x
x
lim
1; lim
1; lim
1; lim
1; lim cos x 1; etc
x 0
x 0
x 0 sin x
x 0 tan x
x 0
x
x

DEPT OF MATHS, SJBIT

Page 54

ENGINEERING MATHEMATICS-I

15MAT11

Evaluate the following limits:


sin x x
Example 1: Evaluate lim
x 0 tan 3 x

lim
x 0

sin x x 0
cos x 1 0
sin x
0
lim
lim
3
2
2
4
3
2
x

0
x

0
tan x 0
3tan x sec x 0
6 tan x sec x 6 tan x sec x 0
lim
x 0

cos x
1

2
4
4
2
6sec x 24 tan x sec x 18 tan x sec x 12 tan x sec x
6
6

Method 2:

sin x x
sin x x 0
x3 0 lim sin x x 0 lim tan x 1
lim

lim
x 0


3
3
x 0 tan 3 x
x 0
x
0
0
tan x 0 x0 x

x
cos x 1 0
sin x 0
cos x
1
lim
lim
lim

2
x 0
3 x 0 x 0 6 x 0 x 0 6
6

ax bx
Example 2: Evaluate lim
x 0
x
ax bx 0

x 0
x 0

lim

a x log a b x log b
x 0
1
a
log a log b log
b
lim

Example 3: Evaluate lim


x 0

lim
x 0

x sin x

x sin x 0
sin x x cos x 0
cos x cos x x sin x 1 1 0 2
lim
lim

1
2
x
x
x

0
x

0
x
2 e x .e x (e x 1)e x 2[1 0] 2
2 e 1 e 0
e 1 0

DEPT OF MATHS, SJBIT

Page 55

ENGINEERING MATHEMATICS-I
x e x log(1 x)
x2

Example 4: Evaluate lim


x 0

lim
x 0

15MAT11

x e x log(1 x) 0
lim
x2
0 x 0

1
1
ex ex x ex
2
1 x 1 1 0 1 3
1 x 0 lim
x 0
2x
2
2
2
0

ex x ex

cosh x cos x
x 0
x sin x

Example 5: Evaluate lim

cosh x cos x 0
sinh x sin x 0
cosh x cos x
11
2
lim
lim

x 0
x sin x
0 x0 sin x x cos x 0 x0 cos x cos x x sin x 1 1 0 2

lim

cos x log(1 x) 1 x
x 0
sin 2 x

Example 6: Evaluate lim

cos x log(1 x) 1 x 0
lim
x 0
sin 2 x
0 x 0

1
1
cos x
1
(1 x)2 1 1
1 x 0 lim

0
x 0
sin 2 x
2 cos 2 x
2
0

sin x

lim

Example 7: Evaluate lim


x 1

lim
x 1

xx x
x 1 log x

xx x 0
x x (1 log x) 1 0
x x (1 log x) 2 x x 1 1 1

lim

lim

2

x1
1
1
x 1 log x 0 x1
0
1

1
x
x2

since y x x log y x log x

1
y
y

1 log x y y (1 log x)
then

d x
( x ) x x (1 log x)
dx

Example 8: Evaluate lim


x

sec2 x 2 tan x
1 cos 4 x

sec x 2 tan x 0
2sec2 x tan x 2sec2 x 0
2sec4 x 4sec2 x tan 2 x 4sec2 x tan x

lim

lim


1 cos 4 x 0 x
4 sin 4 x
16 cos 4 x
0 x
2

lim
x

DEPT OF MATHS, SJBIT

Page 56

ENGINEERING MATHEMATICS-I

2 (1)
2

15MAT11
4

16

Example 9: Evaluate lim


x a

8 1

16 2

log(sin x. cos ec a)
log(cos a.sec x)

cos x cos ec a
sin x. cos ec a
log(sin x. cos ec a ) 0

lim cot x cot 2 a


lim

lim

x a
log(cos a.sec x) 0 xa sec x tan x.cos a xa tan x
cos a.sec x

e x e x 2cos x
x 0
x sin x

Example 10: Evaluate lim

e x e x 2cos x 0
e x e x 2sin x 0
e x e x 2cos x
11 2

lim

lim

x 0
x sin x
0 x0 sin x x cos x 0 x0 cos x cos x x sin x 1 1 0

lim

Example 11: Evaluate lim


x 0

x cos x log(1 x)
x2

x cos x log(1 x) 0
lim
lim
x 0
x2
0 x 0

cos x x sin x

sin x sin x x cos x


lim
x 0

2x

1
(1 x) 2

1
1 x 0

0

0 0 0 1 1

2
2

log(1 x 2 )
x 0 log cos x
2 x
2
1 x 2
log(1 x ) 0
2 x cos x 0
2 cos x 2 x sin x
20
lim
lim
lim
lim

2
2
x 0 log cos x
x

0
x

0
x

0
(1 x )sin x 0
(1 x ) cos x 2 x sin x 1 0
sin x
0

cos x

Example 12: Evaluate lim

tan x sin x
x 0
sin 3 x

Example 13: Evaluate lim

tan x sin x 0
sec2 x cos x 0
2sec2 x tan x sin x 0

lim

lim



x 0
sin 3 x 0 x0 3sin 2 x cos x 0 x0 6sin x cos 2 x 3sin 3 x 0

lim

DEPT OF MATHS, SJBIT

Page 57

ENGINEERING MATHEMATICS-I

15MAT11

4sec2 x tan 2 x 2sec4 x cos x


0 2 1 3 1


3
2
2
x 0 6cos x 12sin x cos x 9 sin x cos x
600 6 2

lim

Method 2:

tan x sin x
tan x sin x
tan x sin x 0
sin x
x3
lim
lim
lim
lim
1

3
3
3
x 0
x 0
x 0
x 0
sin x
x
x
0
sin x

x
sec2 x cos x 0
2sec2 x tan x sin x 0
lim

lim
x 0

x 0
3x 2
6x
0
0
4sec2 x tan 2 x 2sec4 x cos x 0 2 1 3 1
lim


x 0
6
6
6 2

Example 14: Evaluate lim


x 0

tan x x
x 2 tan x

tan x x
tan x x
tan x x 0
tan x
x3
lim 2
lim
lim
lim
1

3
x 0 x tan x
x 0 tan x
x 0
x

0
x
x
0

x
sec2 x 1 0
2sec2 x tan x 0
4sec2 x tan 2 x 2sec4 x 0 2 1
lim

lim

lim


x 0
x 0
3 x 2 0 x 0
6x
6
6
3
0

eax e ax
x 0 log(1 bx )
eax e ax 0
aeax ae ax
lim

lim
x 0
x 0 log(1 bx ) 0
b / (1 bx)

a a 2a

b
b

Example 15: Evaluate lim

a x 1 x log a
Example 16: Evaluate lim
x 0
x2
a x 1 x log a 0
a x log a log a 0
a x (log a) 2 1
lim

lim

lim
(log a)2

2
x 0
x
2x
2
2
0 x 0
0 x 0
e x log(e ex)
x 0
x2

Example 17: Evaluate lim


DEPT OF MATHS, SJBIT

Page 58

ENGINEERING MATHEMATICS-I

15MAT11

e x log(e ex)
e x log e(1 x)
e x log e log(1 x) 0
lim
lim
lim

x 0
x 0
x 0
x2
x2
x2
0
1
1
ex
ex
(1 x)2 1 1
1 x 0 lim
lim

1

x 0
2 x 0 x 0
2
2

Limits of the form :

Example 18: Evaluate lim


x 0

log(sin 2 x)
log(sin x)

log(sin 2 x)
(2cos 2 x / sin 2 x)
2cot 2 x
2 tan x 0
2sec 2 x 2

lim

lim

lim

lim
1
x 0
x 0
x 0 log(sin x)
x 0 cot x
x 0 tan 2 x 0
(cos x / sin x)
2sec2 2 x 2

lim

Example 19: Evaluate lim


x 0

log x
cos ecx

log x
1\ x
sin 2 x 0
2sin 2 x
0

lim

lim
lim

x 0 cos ecx
x0 cos ec x.cot x x0 x cos x 0 x0 cos x x sin x 1 0

lim

Example 20: Evaluate lim


x

lim
x

log cos x
tan x

log cos x
tan x
tan x
sin x cos x 0
lim
lim

0
lim
2
2

tan x x sec x
1
1
x sec x
x
2

log(1 x)
x 1 cot x

Example 21: Evaluate lim

log(1 x)
1/(1 x)
sin 2 x 0
2 sin x cos x
0

lim

lim
lim

2
x 1 cot x

x1 cos ec x x1 (1 x) 0 x1

lim

Example 22: Evaluate lim log tan 2 x tan 3x


x 0

log tan 3x
lim log tan 2 x tan 3x lim

x 0 log tan 2 x
x 0

DEPT OF MATHS, SJBIT

logb a

log e a
log e b

Page 59

ENGINEERING MATHEMATICS-I

15MAT11

3sec2 3x / tan 3x
3/ sin 3x.cos 3x
3/ sin 3x.cos 3x
lim
lim
lim

2
x 0 2sec 2 x / tan 2 x

x0 2 / sin 2 x.cos 2 x x0 2 / sin 2 x.cos 2 x

6 / sin 6 x
6sin 4 x 0
24cos 4 x 24
lim
lim
lim
1

x 0 4 / sin 4 x

x0 4sin 6 x 0 x0 24cos 6 x 24

Example 23: Evaluate lim


x a

log( x a)
log(e x ea )

log( x a)
1/( x a)
(e x e a ) 0
ex
ea

lim

lim

lim

1
x a x x a

x a log(e x e a )
e /(e e ) xa e x ( x a) 0 xa e x ( x a) e x e a

lim

Limits of the form 0 : To evaluate the limits of the form 0 , we rewrite the
0
given expression to obtain either or form and then apply the LHospitals Rule.
0
1

Example 24: Evaluate lim(a x 1) x


x

1
a x (log a) 2
1
(a 1) 0
x
lim(a x 1) x 0 form lim
lim
x
x 1
x

1

0

2
x
x
1

1
x

lim a x (log a) a 0 log a log a


x

Example 25: Evaluate lim(1 sin x) tan x


x

lim(1 sin x) tan x 0 form lim


x

lim
x

cos x
0
0
2
cos ec x 1

Example 26: Evaluate limsec


x 1

limsec
x 1

(1 sin x) 0

cot x 0

2x

2x

.log x

.log x 0 form lim

DEPT OF MATHS, SJBIT

x 1

log x 0

cos 0
2x

Page 60

ENGINEERING MATHEMATICS-I
1/ x

lim
x

15MAT11

1
sin 2
2
2 x x

lim
x 1

2x

sin

2x

Example 27: Evaluate lim x log tan x


x 0

lim x log tan x 0 form lim


x 0

x 0

log tan x

(1/ x)

sec2 x / tan x
x2
lim
x 0
x 0 sin x.cos x
1
2
x

lim

2 x 2 0
4 x
0
lim
0

x 0 sin 2 x 0
x0 2cos 2 x 2

lim

Example 28: Evaluate lim (1 x 2 ) tan


x 1

x
2

0 form
2
1 x2 0
2 x
lim
lim

x 1
x

1
x 0

x
cot
cos ec 2
2
2
2
2
4

lim (1 x 2 ) tan
x 1

Example 29: Evaluate lim tan x.log x


x 0

lim tan x.log x 0 form lim


x 0

x 0

log x

cot x

1/ x
sin 2 x 0
sin 2 x 0
lim
lim
0
lim
x 0 cos ec 2 x
x 0
x

0
x 0
1
1

Limits of the form : To evaluate the limits of the form , we take


0
L.C.M. and rewrite the given expression to obtain either or form and then apply
0
the LHospitals Rule.

DEPT OF MATHS, SJBIT

Page 61

ENGINEERING MATHEMATICS-I

15MAT11

Example 30: Evaluate lim cot x


x 0 x

1 cos x
lim cot x lim
form
x 0 x

x0 x sin x

sin x x cos x 0
cos x cos x x sin x
lim
lim

x 0
x

0
x sin x
sin x x cos x

0
x sin x

0
lim

x 0 sin x x cos x 0


sin x x cos x

lim
x 0 cos x cos x x sin x

00

0
11 0
Example 31: Evaluate lim sec x tan x
x

sin x
1
lim sec x tan x lim

form

cos x
cos x
x
x
2
2
1 sin x 0
cos x 0
lim
lim

1 0

x cos x 0
x sin x
2
2
1
x
Example 32: Evaluate lim

x 1 log x
x 1

1
( x 1) x log x 0
x
lim

form lim


x 1 log x
x 1
x 1

( x 1) log x 0

1 1 log x
log x 0
l/ x 1 1
lim

lim
1
lim
1 1
x 1 x 1
x 1
x 1
0
1

1
2

log x
1 log x

2
x
x

DEPT OF MATHS, SJBIT

Page 62

ENGINEERING MATHEMATICS-I

15MAT11

1
1
Example 33: Evaluate lim x
x 0 x
e 1

(e x 1) x 0
1
1
lim x form lim

x
x 0 x
x 0
e 1

x(e 1) 0

ex 1
ex
lim x

lim

x
x
x
x
x 0 (e 1) xe

0 x0 e e xe
1
1

11 0 2

1
1
Example 34: Evaluate lim

x 0 sin x
x

1
1
x sin x 0
lim
form lim

x 0 sin x
x 0
x

x sin x 0
sin x
0
1 cos x 0

lim
lim

x 0 sin x x cos x
x 0 cos x cos x x sin x

11
1 log(1 x)
Example 35: Evaluate lim

x 0 x
x2

1 log(1 x)
x log(1 x) 0
lim
lim
2

0
x 0 x
x
x2

x 0
1
1

(1 x) 2 1

0
lim 1 x lim

x 0
x 0
2
x
0
2

x
a
Example 36: Evaluate lim cot
x 0 x
a

x
x

a cos a
a cos a 0
x
a
lim cot lim
form xlim

x 0 x
0
x
a x0 x sin x

x sin 0
a
a

DEPT OF MATHS, SJBIT

Page 63

ENGINEERING MATHEMATICS-I

15MAT11

x
x
x
x x
x

1
a sin a x cos a 0
a. a cos a cos a a sin a
lim
xlim

x 0
0
x
x x
x

x sin
sin cos
a
a a
a

x
x
1
x x 1
x

sin
sin . .cos

0
00

a
a
a
a a a
a
lim
lim

x 0
x 0
x x
x
1
x 1
x x
x
1 1
sin cos 0
cos cos 2 sin
0
a a
a
a a
a a
a a a

a
sin 2 x a sin x
is finite. Also find the
x 0
x3

Example 37: Find the value of a such that lim


value of the limit.
Let A= lim
x 0

sin 2 x a sin x 0
2cos 2 x a cos x 2 a

finite
lim
3
x

0
x
3x 2
0
0

We can continue to apply the LHospitals Rule, if 2+a=0 i.e., a= -2 .


For a 2 ,
2cos 2 x 2cos x 0
4sin 2 x 2sin x 0
A lim
lim


2
x 0
3x
6x
0 x 0
0

8cos 2 x 2cos x 8 2

1
x 0
6
6

lim

The given lim it will have a finite value when a 2 and it is 1.

Example 38: Find the values of a and b such that lim


x 0

Let A lim
x 0

x(1 a cos x) b sin x 1


.
x3
3

x(1 a cos x) b sin x 0


(1 a cos x) ax sin x b cos x 1 a b

finite
lim
3
x
3x 2
0
0 x 0

We can continue to apply the LHospitals Rule, if 1-a+b=0 i.e., a-b = 1 .


For a b 1 ,

A lim
x 0

(1 a cos x) ax sin x b cos x 0


2a sin x ax cos x b sin x 0
lim

2
x

0
3x
6x
0
0

DEPT OF MATHS, SJBIT

Page 64

ENGINEERING MATHEMATICS-I

15MAT11

3a cos x ax sin x b cos x 3a b

finite
x 0
6
6

lim

1
3a b 1
This finite value is given as . i.e.,
3a b 2
3
6
3
Solving the equations a b 1 and 3a b 2we obtain a

1
1
and b .
2
2

a cosh x b cos x
1.
x 0
x2

Example 39: Find the values of a and b such that lim

a cosh x b cos x a b

finite
x2
0
We can continue to apply the LHospitals Rule, if a-b=0, since the denominator=0 .
Let A lim
x 0

For a b 0 ,

a cosh x b cos x 0

x 0
x2
0

A lim

a sinh x b sin x 0

2x
0
a cosh x b cos x a b
lim

x 0
2
2
lim
x 0

But this is given as 1.


a b 2
Solving the equations a b 0 and a b 2 we obtain a 1 and b 1.

Limits of the form 00 , 0 and 1 : To evaluate such limits, where function to the
power of function exists, we call such an expression as some constant, then take
0

logarithm on both sides and rewrite the expressions to get or


form and then apply
0

the LHospitals Rule.


Example 40: Evaluate lim x x
x 0

Let A lim x x (00 form)


x 0

Take log on both sides to write


DEPT OF MATHS, SJBIT

Page 65

ENGINEERING MATHEMATICS-I

15MAT11
log x

x 0 1/ x

log e A lim log x x lim x.log x (0 form) lim


x 0

x 0

lim
x 0

1/ x
x 0
lim
0
2
x

0
(1/ x )
1 1

loge A 0 A e0 1 lim x x 1
x 0

Example 41: Evaluate lim(cos x)

x2

x 0

Let A lim(cos x)

x2

x 0

(1 form)

Take log on both sides to write


1

log e A lim log(cos x)

lim

x2

x 0

x 0

1
log cos x 0
log cos x ( 0 form) lim

2
x

0
x
x2 0

tan x 0
sec2 x 1

lim


x 0
2 x 0 x 0
2
2

lim

1
1
1
log e A A e 2
2
e

lim(cos x) x
2

x 0

1
e

Example 42: Evaluate lim(tan x)cos x


x

Let A lim(tan x)cos x (0 form)


x

Take log on both sides to write


log e A lim log(tan x)cos x lim cos x log(tan x) (0 form)
x

lim
x

log tan x
sec2 x / tan x
cos x 0

lim
lim 2 0

sec x x sec x.tan x x sin x 1
2

log e A 0 A e0 1 lim(tan x)cos x 1


x

DEPT OF MATHS, SJBIT

Page 66

ENGINEERING MATHEMATICS-I

15MAT11

tan x x2
Example 43: Evaluate lim(
)
x 0
x
1
tan x x2
Let A lim(
) (1 form)
x 0
x

Take log on both sides to write


1
tan x x2
1
tan x
log e A lim log(
) lim 2 log(
) ( 0 form)
x 0
x

0
x
x
x
tan x
sec 2 x 1
log(
)

0
x
tan
x
x
lim
lim
x 0
x 0
x2
0
2
x

1
1
2
1

2 x sin 2 x 0
lim sin x.cos x x lim sin 2 x x lim 2

x 0
x

0
x

0
2x
2x
2 x sin 2 x 0
lim
x 0

2 2cos 2 x
4sin 2 x
0
0
lim

2
2
x

0
4 x sin 2 x 4 x cos 2 x 0
4sin 2 x 16 x cos 2 x 8 x sin 2 x 0

8cos 2 x
8 1

2
x 0 24cos 2 x 48 x sin 2 x 16 x cos 2 x
24 3

lim

1
1

1
tan x x2
log e A A e 3 lim(
) e 3
x 0
3
x
1

Example 44: Evaluate lim(a x x) x


x 0

Let A lim(a x x) x (1 form)


x 0

Take log on both sides to write


1
1
log e A lim log(a x x) x lim log(a x x) ( 0 form)
x 0
x 0 x
x
log(a x) 0
(a x log a 1) /(a x x)
lim

lim
x 0
x 0
x
1
0

log a 1 log a log e log ae

log e A log ea A ea

DEPT OF MATHS, SJBIT

Hence lim(a x x) x ea.


x 0

Page 67

ENGINEERING MATHEMATICS-I

15MAT11

x tan x
Example 45: Evaluate lim(2 ) 2 a
x a
a
x tan 2 ax
Let A lim(2 )
(1 form)
x a
a
Take log on both sides to write
x tan 2 ax
x
x
log e A lim log(2 )
lim tan
.log(2 ) 0 form
x a
x

a
a
2a
a
(1/ a)

x
x
x
log(2 )
2
sin 2

0
2
a lim
a
2a 2
lim .
lim

x a
x 0 x a
x x a
x

cot
cos ec 2
2
2a
2a
2a
a

log e A

Ae

2
x tan 2 ax

Hence lim(2 )
e .
x a
a

PARTIAL DIFFERENTIATION:
Introduction: We often come across qualities which depend on two or more variables.
For e.g. the area of a rectangle of length x and breadth y is given by
Area = A(x,y) = xy. The area A(x, y) is, obliviously, a function of two variables.
Similarly, the distances of the point (x, y, z) from the origin in three-dimensional space is
an example of a function of three variables x, y, z.
Partial derivatives: Let z = f(x, y) be a function of two variables x and y.
f
z
The first order partial derivative of z w.r.t. x, denoted by
or
or zx or fx is defined
x
x
z
f x x, y f x, y
as
lim

0
x
x
z
From the above definition, we understand that is the ordinary derivative of z
x
w.r.t x, treating y as constant.
z
f
The first order partial derivative of z w.r.t y, denoted by
or
or zy or fy is defined as
y
y

DEPT OF MATHS, SJBIT

Page 68

ENGINEERING MATHEMATICS-I

15MAT11

z
f x, y y f x, y
lim
y y 0
y

z
From the above definition, we understand that is the ordinary derivative of z
y
w.r.t y, treating x as constant
z 2 z
2 f
The partial derivatives 2 or
or zxx or fxx;
x 2
x x x
z 2 z
2 f
or zyy or fyy;
2 or
y y y
y 2
z 2 z
or zyx or fyx

y y xy
z 2 z
or zxy or fxy

y y xy
are known as second order Partial derivatives.
and

In all ordinary cases, it can be verified that


2z
2z

xy yx
The third and higher order partial derivatives of f(x,y) are defined in an analogous
way Also, the second and higher order partial derivatives of more than two
independent variables are defined similarly.
A note on rules of partial differentiation:All the rules of differentiation applicable to functions of a single
independent variable are applicable for partial differentiation also; the only
difference is that while differentiating partially w.r.t one independent variable all
other independent variables are treated as constants.

Total derivatives, Differentiation of Composite and Implicit functions


In this lesson we learn the concept of total derivatives of functions of two or
more variables and, also rules for differentiation of composite and implicit functions.
a) Total differential and Total derivative:For a function z f ( x, y) of two variables, x and y the total differential (or
exact differential ) dz is defined by:
f
f
dz
dx dy --------------------------(1)
x
x
Further, if z f ( x, y) where x x(t ) , y y(t ) i.e. x and y are themselves
functions of an independent variable t, then total derivative of z is given by
DEPT OF MATHS, SJBIT

Page 69

ENGINEERING MATHEMATICS-I

15MAT11

dz f dx f dy
-------------------------(2)

dt x dt y dt
Similarly, the total differential of a function u f ( x, y, z) is defined by
f
f
f
du
dx dy dz -----------------(3)
x
y
z
Further, if u f ( x, y, z) and if x x(t ) , y y(t ) , z z (t ) , then the total
derivative of u is given by
du f dx f dy f dz
---------------(4)

dt x dt y dt z dt

(b) Differentiation of implicit functions:An implicit function with x as an independent variable and y as the dependent
variable
is
generally
of
the
form z f ( x, y) 0 .
This
gives
dz df
0 .Then, by virtue of expression (2) above, we get
dx dx

dz f f dy
df f f dy
or
, and hence

dx x y dx
dx x y dx
f
x
dy
f f dy
, so that we get
----------- (5)

0

dx
x y dx
f
y
(c) Differentiation of composite functions:Let z be an function of x and y and that x (u, v) and y (u, v) are functions
of u and v then,

z f x f y

u x u y u
&

z f x f y

v x v y v

-------------------(6)

Similarly, if z f (u, v) are functions of u and v and if u ( x, y) and


v ( x, y) are functions of x and y then,

DEPT OF MATHS, SJBIT

Page 70

ENGINEERING MATHEMATICS-I

15MAT11

z f u f v

x u x v x

&

z f u f v

y u y v y

--------------------------(7)

Note:-1) The above formulae can be extended to functions of three are more
variables and formulas (6) and(7) are called Chain rule for partial differentiation.
2) The second and higher order partial derivatives of z f ( x, y) can be obtained
by repeated applications of the above formulas

Evaluate:
1. Find the total differential of
(i) e x x sin y y cos y
(ii) e xyz
Sol:- (i) Let z f x, y e x x sin y y cos y Then
z
e x 1 x sin y y cos y
x
z
and
e x 1 x cos y y sin y Hence, using formula (1), we get
y
z
z
dz dx dy
x
y
x
i.e dz e 1 x sin y y cos ydx e x 1 x cos y y sin ydy
(ii) Let z f ( x, y, z ) e xyz Then
u
u
u
yz e xyz ;
xy e xyz
xz e xyz ;
x
z
y
Total differential of z f ( x, y, z ) is (see formula (3) above)

u
u
u
du
dx
dy
dz
x
y
z
e xyz yzdx zxdy xydz
dz
2. Find
if
dt
(i) z xy 2 x 2 y ,where x at 2 , y 2at
(ii) u tan 1 y ,where x e t e t , y e t e t
(VTU-Jan 2003)
x
Sol:- (i) Consider z xy 2 x 2 y
z
z
y 2 2 xy &
2 xy x 2
x
y

DEPT OF MATHS, SJBIT

Page 71

ENGINEERING MATHEMATICS-I
Since x at 2 & y 2at , We have

15MAT11

dx
dy
2at ,
2a
dt
dt

Hence, using formula (2), we get


dz z dx z dy

dt x dt y dt
y 2 2 xy 2at 2 xy x 2 2a
y 2 2 xy y 2a 2 xy x 2 , Using y 2at
dz
y 3 2 xy 2 4axy 2ax 2
dt
dz
To get explicitly in terms of t, we substitute
dt
2
x at & y 2at , to get
dz
3
3
4
2a 8t 5t
dt

(ii) Consider

u tan 1 y
x
u
y
u
x
,
2
2
2
x x y y x y 2
Since x e t e t & y e t e t ,we have
dx
dy
e t e t y
e t e t x
dt
dt
du u dx u dy
Hence
( see eqn (2))

dt x dt y dt

x2 y2
y

y 2
x 2
2
2
2
2
x y
x y
x y
Substituting x e t e t & y e t e t , we get
du
2
2t
dt e e 2t
dy
3. Find if (i) x y y x =Constant
dx
(ii) x e y 2 xy
Sol: - (i) Let z f ( x, y) x y y x =Constant. Using formula (5)
f
dy
x -----------------(*)
f
dx
y

DEPT OF MATHS, SJBIT

Page 72

ENGINEERING MATHEMATICS-I
But

15MAT11

f
f
yx y 1 y x log y and
x y log x xy x1 Putting those in(*), we get
x
y

yx y 1 y x log y
dy
y
x 1
dx
x log x xy
(ii) Let z f ( x, y) e x e y 2 xy =Constant
f
f
Now,
ex 2y ;
e y 2 x Using this in (8),
x
y
f
e x 2 y
dy
x

y

dx
e 2x
f
y
4. (i) If z f ( x, y) ,where x r cos , y r sin show that
2

1 z
z z
z
2
r
x y
r
2

(VTU July-2005)

(ii) If z f ( x, y) ,where x e u e v & y e u e v ,Show that


z z
z
z

x y
u v
x
y
Sol: As x r cos and y r sin , we have
x
x
y
y
cos ,
r sin ;
sin &
r cos .
r

Using Chain rule (6) & (7) we have


z
z z x z y z

cos sin

y
r x r y r x
z
z z x z y z

r sin r cos

y
x y x

Squaring on both sides, the

above equations, we get


2

z
z
z
z z
2
2
cos sin 2 sin cos
r
x
x y
y
2

z
1 z
z
z z
2
2
cos sin 2 sin cos
2
r
x
x y
y
2

Adding the above equations , we get


DEPT OF MATHS, SJBIT

Page 73

ENGINEERING MATHEMATICS-I
1
z
2
r
r
2

15MAT11

2
2
2

z z

z
2
2
cos sin
x
y

z z
= as desired.
x y
(ii) As x e u e v & y e u e v , We have
x
x
y
y
eu ,
e v ,
e u &
e v
u
v
u
v
Using Chain rule (6) we get
z z x z y z u z v

e e

y
u x u y u x
z
z z x z y z

e v e v

y
v x v y v x
2

z u
z z z u

e e v
e ev

y

z
z

x
y
x
y
u
u
u
5. (i) If u f ( x, z, y / z) Then show that x
y
z
0
x
y
z
(ii) If H f ( x y, y z, z x) , show that
H H H

0
x
y
z
Sol: - (i) Let u f (v, w) , where v xz and w y
z
v
v
w
v
w 1 w y
,
z,
x &
0,

0,

z z
z2
x
z
x
y
y
Using Chain rule,
u u v u w u
z u 0 z u

x v x w x v
w
v

(VTU-July-2004)

(VTU-July-2003)

u u v u w u
0 u 1 z 1 u

y v y w y v
w
z w
u u v u w u
x u y z 2 x u y2 u

z v z w z v
w
v z w
From these, we get
u
u
u
u y u
u y u
x
y
z
xz

z x

x
y
z
v z w v z 2 w
=0
(ii) Let H f (u, v, w) Where u x y, v y z, w z x
DEPT OF MATHS, SJBIT

Page 74

ENGINEERING MATHEMATICS-I

15MAT11

u
u
u
1,
1,
0
x
y
z
v
v
v
0, 1, 1
x
y
z
w
w
u
1,
0,
1 Using Chain rule,
x
y
z
H H u H v H w H
1 H 0 H 1

x
u x v x w x u
v
w
H H u H v H w H
H
1 1 v 0

y
u y v y w y u
v
w
H H u H v H w H
0 H 1 H 1

z
u z v z w z
u
v
w
u
u
u
Adding the above equations, we get the required result x
y
z
0
x
y
z
Now,

Applications to Jacobians:
In this lesson, we study Jacobians, errors and approximations using the
concept of partial differentiation.

Jacobians:Jacobians were invented by German mathematician C.G. Jacob Jacobi (18041851),who made significant contributions to mechanics, Partial differential equations and
calculus of variations.
Definition:- Let u and v are functions of x and y, then Jacobian of u and v w.r.t x and
denoted by
u, v
u, v
or
J or J
x, y x , y
is defined by

u
x
u, v

J
v
x, y
x

u
y
v
y

Similarly, if u, v, w are functions of three independent variables of x, y, z, then

DEPT OF MATHS, SJBIT

Page 75

ENGINEERING MATHEMATICS-I

15MAT11

u u u
x y z
u, v, w v v u

J J
x, y, z x y z
w w w
x y z
Remark:- In a similar way, Jacobian of n functions in n-variables can be defined
Note:- (i) If J

u, v
, then the "inverse Jacobian" of the Jacobian J,
x, y

denoted by J ,is defined as

x, y
u, v

(ii) Similarly, "inverse Jacobian" of J

u, v, w
x, y , z
is defined as J
x, y , z
u, v, w

Properties of Jacobians :Property 1:- If J

u, v
x, y
and J
then JJ 1
x, y
u, v

Proof:- Consider
u
u, v x, y x
JJ 1

x, y u, v v
x

u x
y u

v y
y u

u x u y

x u y u

v x v y

x u y u

x
v
y
v

u x u y

x v y v
v x v y

x v y v

1 0
1 0

Property 2:- (Chain rule for Jacobians):- If u and v are functions of r&s and r,s
are functions x&y,then
u, v
u, v r , s
J

J
J
r , s x, y
x, y

DEPT OF MATHS, SJBIT

Page 76

ENGINEERING MATHEMATICS-I

15MAT11

Proof:- Consider
u
u, v r , s r

J
J
r , s x, y v
r

u r
s x

v s
s x

r
y
s
y

u r u s u r u s

r x s x r y s y

v r v s v r v s

r x s x r y s y
u
x

v
x

u
y
u, v

J
v
x, y
y

Jacobians in various co-ordinate systems:1. In Polar co-ordinates, x r cos , y r sin


u, v
we have
r
r ,
2. In spherical coordinates, x cos , y sin , z z, we have
x, y , z

, , z
3. In spherical polar co-ordinates, x r sin cos , y r sin sin , z r cos
x
y
Proof of 1:- we have,
cos and
sin
r
r
x
y
r sin and
r cos

x x
cos r sin
x, y r

sin r cos
r , y y
r
r cos 2 r sin 2 r cos 2 sin 2 r

y
x
z
sin ,
cos ,
0

x
y
z
sin ,
cos ,
0

Proof of 2 :-we have

DEPT OF MATHS, SJBIT

Page 77

ENGINEERING MATHEMATICS-I

15MAT11

z
z
z
0, 0
0,
z

x, y, z y

, , z
z

x
z
cos sin 0
y
sin cos
0
z
0
0
1
z
z

Proof of 3:- We have


x
x
x
sin cos ,
r cos cos ,
r sin sin
r

y
y
y
sin sin ,
r cos sin ,
r sin cos
r

z
z
z
cos ,
r sin ,
0
r

sin cos r cos cos r sin sin


x, y , z

sin sin r cos sin r sin cos


r , ,
cos
r sin
0
r 2 sin

Evaluate
1. If u x 2 2y 2 , v 2 x 2 y 2 , where
x r cos , y r sin show that
u, v
6r 3 sin 2
r ,
Consider u x 2 2 y 2 r 2 cos 2 2r 2 sin 2

(VTU-Jan-2006)

v 2 x 2 y 2 2r 2 cos 2 r 2 sin 2
v
u

2r cos 2 4r sin 2 ,
4r cos 2 2r sin 2
r
r
u
2r 2 cos sin 4r 2 sin cos

v
4r 2 cos sin 2r 2 sin cos

u u
2r cos 2 4r sin 2 2r 2 cos sin 4r 2 sin cos
u, v r

r , v v
4r cos 2 2r sin 2 4r 2 cos sin 2r 2 sin cos
r

DEPT OF MATHS, SJBIT

Page 78

ENGINEERING MATHEMATICS-I

15MAT11

2r cos sin 4r sin cos 4r cos

2r cos 2 4r sin 2 4r 2 cos sin 2r 2 sin cos


2

2r sin

6r sin 2
3

x, y 1 x, y
2. If x u1 v , y uv, Prove that J
J
1
u, v
u, v
x
x
Consider
1 v ,
u
u
v
y
y
v,
u
u
v
x x
x, y u v 1 v u
J

u
u, v y y v
u v
1 v u uv u uv uv u

(VTU-2001)

x, y
J
u (1)
u, v
Further, as x u1 v , y uv,
u uv
We write, x u y u x y and
y
y y


v
u x y
x y
u
u

1,
1 and
x
y
v
y
v
x

,
2
x
x y y x y 2
u u
1
1
x y
1 u, v

J

y
x

2
x, y v v
x y x y 2
x y
v

x y

x y

1 1


x y u

JJ 1 u

1
1
u

3. If x e u cos v, y e u sin v, Prove that


x, y u, v

1
u, v x, y
Consider

x e u cos v

DEPT OF MATHS, SJBIT

y e u sin v

Page 79

ENGINEERING MATHEMATICS-I

15MAT11

y
x
e u cos v
e u sin v
u
u
y
x
e u sin v
e u cos v
v
v
x x
u
u
x, y u v e cos v e sin v

u, v y y eu sin v eu cos v
u v
x, y
i.e
e 2u (1)
u, v
Again Consider x e u cos v, y e u sin v,
1
x 2 y 2 e2u or u log x 2 y 2
2
y
& tan v or v tan 1 y
x
x
u
x
u
y
Hence
2
,;
2
,;
2
y x y 2
x x y
v
y
v
x
&
2
;
2
2
x x y y x y 2
u u
x
y
2
2
2
x y
x y2
u, v x y
1

2
x2 y 2
2
y
x
x, y v v
x y
2
2
2
2
x y
x y
x y
u, v
i.e
e 2u (2)
x, y
x, y u, v

e2u e 2u 1
u, v x, y
yz
zx
xy
u, v, w
4. If u , v , w , Show that
4
x, y , z
x
y
z

u u
x y
u, v, w v v

Now,
x, y, z x y
w w
x y

DEPT OF MATHS, SJBIT

u
yz z y
2
z
x
x x
v
z zx x

z
y
y2
y
w
y x xy
z
z z
z2

Page 80

ENGINEERING MATHEMATICS-I

i.e

15MAT11

u, v, w yz zx xy z z xy y x




x, y, z x 2 y 2 z 2 x y z 2 z y
y z x y zx
2
x y z z y
= 4, as desired.

x, y, z
r 2 sin

r, ,

5. If x r sin cos , y r sin sin , z r cos ,show that


Now, by definition

x
r
x, y, z y

r , , r
z
r

i.e

sin cos r cos cos


x, y , z
sin sin r cos sin
r , ,
cos
r sin

r sin sin
r sin cos

sin cos 0 r 2 sin 2 cos

r cos cos 0 r sin cos cos

r sin sin r sin 2 sin r cos 2 sin

r 2 sin 2 sin cos 2 r 2 sin cos 2 cos 2

sin cos

r 2 sin sin 2 cos 2 cos 2 r 2 sin sin 2


r2

sin 2

r 2 sin , as required

DEPT OF MATHS, SJBIT

Page 81

ENGINEERING MATHEMATICS-I

15MAT11

MODULE III
VECTOR CALCULUS
CONTENTS:
Vector function83
Velocity and Accleleration..87
Gradient, Divergence, Curl, Laplacian Vector function.92
Solenoidal and Irrotational vectors..94
Vector Identities..........99

DEPT OF MATHS, SJBIT

Page 82

ENGINEERING MATHEMATICS-I

15MAT11

Introduction:
Basically vector is a quantity having both magnitude and direction. Vector
quantities like force, velocity, acceleration etc. have lot of reference in physical and
engineering problems. We are familiar with vector algebra which gives an exposure to all
the basic concepts related to vectors.
Differentiation and Integration are well acquainted topics in calculus. In the
background of all these we discuss this chapter vector calculus comprising vector
Differentiation. Many concepts are highly significant in various branches of engineering.

Basic Concepts Vector function of a single variable and the derivative


of a vector
Let the position vector of a point p(x, y, z) in space be

r xi yi zk

If x, y, z are all functions of a single parameter t, then r is said to be a vector function of



t which is also referred to as a vector point function usually denoted as r = r (t). As the
parameter t varies, the point P traces in space. Therefore

r = x(t) i+y (t) j+z(t) k


is called as the vector equation of the curve.

dr
dx dy
dz
r ' (t ) i
j k
dt
dt
dt
dt
Is a vector along the tangent to the curve at P.
If t is the time variable,

dr

gives the velocity of the particle at time t.


dt

d d dr d 2r

Further a
represents the rate of change of velocity
dt dt dt dt 2

and is called the acceleration of the particle at time t.
1.

d
c1 r1 (t ) c2 r2 (t ) c1 r1'(t ) c2 r2 (t ) where c1 , c2 are constants.
dt

2. d F . G F . d G d F . G
dt
dt
dt

DEPT OF MATHS, SJBIT

Page 83

ENGINEERING MATHEMATICS-I

15MAT11

d dG d F
F G F .

G
3. dt
dt
dt

Where F F (t ) and G G (t ).

Gradient, Divergence, Curl and Laplacian:

If is scalar function A is vector function A a1i a2 j a3k then

1. ie., grad
i
j
k
x
y
z

2. If A a1i a2 j a3k ,


div A . A i
j k . a1i a2 j a3k
z
x y

a a a
div A . A 1 2 3
x y
z

3.

If A a1i a2 j a3k ,
i

div A A
x

a1

j k


.
y z

a2 a3
a
a a
a2 a3 a1
i 3

k 2 1
j

z x
z
y
y
x

2 2 2

x 2 y 2 z 2

4.

Laplacian of 2

5.

2 A 2 A 2 A
Laplacian of A A 2 2 2
x
y
z

Important points:

d2 r
dr

1. If r x(t )i y (t ) j z (t )k , then
is velocity and
is acceleration.
dt 2
dt

DEPT OF MATHS, SJBIT

Page 84

ENGINEERING MATHEMATICS-I

15MAT11


dT
d r
dt
and unit normal vector is n ds where
2. The unit tangent vector T

T
d r
dt

ds
T .
dt

3. If A and B are any two vector and is angle between two vectors, then

A.B
cos 1 .
A. B

4. Component of a vector (velocity or acceleration) F along a given vector C is the

resolved part of F given by F . n where n

5. Component of a vector F along normal to the

is given by


F resolved part of acceleration along c F F . c .c .

1.

Find the unit tangent vector to the curve r cos ti sin tj tk.

Soln: Given the space curve r cos ti sin tj tk

dr
T
sin ti cos tj k
dt

T cos 2 t sin 2 t 1 1 1 2

Therefore, the unit tangent vector to the given curve at any point is

sin ti cos tj k
T
1
T

sin ti cos tj k
2
2
T

2.

Find the unit normal vector to the curve r 4sin ti 4cos tj 3tk.

Soln: Given r 4sin ti 4cos tj 3tk

DEPT OF MATHS, SJBIT

Page 85

ENGINEERING MATHEMATICS-I

15MAT11

dr
T
4 cos ti 4sin tj 3k
dt

T 16 cos 2 t sin 2 t 9 25 5

Therefore, the unit tangent vector to the given curve at any point t is

4 cos ti 4sin tj 3k
T
1

T
4 cos ti 4sin tj 3k
5
5
T

dT

ds

dT
dt
ds
dt

dT

dt 1 1 4sin ti 4 cos tj


5 5

dr
dt

dT
4

sin ti cos tj
ds
25

2
dT
4
4
and
sin 2 t cos 2 t
ds
25
25

The unit normal vector to the given curve is


dT

ds (4 / 25) sin ti cos tj

sin ti cos tj

(4
/
25)
dT

ds

3.

Find the angle between tangents to the curve x t , y t , z t at t=2 and


t=3.
2

Soln: Define the position vector r x(t )i y (t ) j z (t )k

r r 2i t 3 j t 4 k

i.e.,

dr
T
2ti 3t 2 j 4t 3 k
dt

AT

DEPT OF MATHS, SJBIT

t 2

4i 12 j 32k 4(i 3 j 8k)

Page 86

ENGINEERING MATHEMATICS-I

15MAT11

A 16(1 9 64) 4 74

B T

t 3

6i 27 j 108k 3(2i 9 j 36k)

9(4 81 1296) 3 1381

Let be the angle between two vectors A and B , then

cos 1

cos 1

30

4.


4 i 3 j 8k .3 2i 9 j 36k
A. B
1

cos

4 74 3 1381
A . B

2 27 248
1
cos 0.8665
74 1381

A particle moves along the curve x 1 t , y 1 t , z 2t 5. Determine its


velocity and acceleration. Find the component of velocity and acceleration at t=1 in
the direction 2i j 2k
Soln: Given the position vector
3

r (1 t 3 )i (1 t 2 ) j (2t 5)k

dr
the velocity , v
(3t 2 )i (2t ) j (2)k
dt

d2 r
and acceleration a
(6t 3 )i 2 j
dt 2
at t 1,

v 3i 2 j 2k

a 6i 2 j
Therefore, the component of velocity vector in the given direction

a 2i j 2k.

(2i j 2k) 6 2 4
v . n (3i 2 j 2k).

0
3
4 1 4
The normal component of acceleration in the given direction

(2i j 2k) 12 2
10
a . n (6i 2 j ).

.
3
3
4 1 4

DEPT OF MATHS, SJBIT

Page 87

ENGINEERING MATHEMATICS-I
5.

15MAT11

A particle moves on the curve x 2t 2 , y t 2 4t , z 3t 5, where t is the time.


Find the components of velocity and acceleration at time t=1 in the direction
i 3 j 2k.

Soln: The position vector at any point (x,y,z) is given r xi yj zk , but

r 2t 2i (t 2 4t ) j (3t 5)k
Therefore, the velocity and acceleration are

dr
v
4ti (2t 4) j 3k
dt

d2 r
a
4i 2 j
dt 2

at t 1, v 4i 2 j 3k

a 4i 2 j

Therefore the component of velocity in the given direction i 3 j 2k is

(i 3 j 2k) 16
v . n (4i 2 j 3k).

1 9 4
14
Since dot product of two vector is a scalar.
The components of acceleration in the given direction i 3 j 2k is

(i 3 j 2k) 2
a . n (4i 2 j ).

1 9 4
14

Unit Normal Vector:

n
n
n

where n Normal vector

Directional Derivative (D.D.):

If a is any vector and is any scalar point function then Directional Derivative

.a
(D.D.)= .a
a

Maximum directional derivative (Normal derivative)

The directional derivative will be the maximum in the direction

(i.e., a )

=
= .

and the maximum value of directional derivative = .


DEPT OF MATHS, SJBIT

Page 88

ENGINEERING MATHEMATICS-I

15MAT11

Note : The maximum directional derivative is also called normal derivative i.e., Normal
derivative =

Equations of Tangent plane and Normal Line:

( x, y, z ) c be any given surface and ( x1 , y1 , z1 ) be a point on it, then


(i) Equation of tangent plane to ( x, y, z ) c at P( x1 , y1 , z1 ) is
Let



(x x )
(y y )

1 x
1 y
p


( z z1)
z
p

0
p

(ii) Equation of normal line to ( x, y, z) c at P( x1 , y1 , z1 ) is

x x1 y y1 z z1


Ex. 1: If x3 y3z3 , find at (1,2,1) along i+2j+2k
Soln: Given x3 y3 z3 , then


i+
j+
k
x y
z
x3 y 2 z 3 )j (3x3 y3 z 2 )k
=(3x 2 y3 z 3 )i+(3

(1,2,1) 24i 12j 24k

Let a i 2j 2k a
1 4 4 3

i 2j 2k
3

at (1,2,1) along the vectors a is


i 2j 2k

. a = 24i 12 j 24k .
3
1
96
(24 24 48)
3
3

. a = 32
DEPT OF MATHS, SJBIT

Page 89

ENGINEERING MATHEMATICS-I

15MAT11

Ex. 2: Find the unit normal vector to the surface xy z 4 at the point
(-1,-1,2).
xy 3 z 2

x 2 z 2j 2 xy3 zk
Soln: Let = i+ j+ k y3 z 2 )i+3
x y
z
and
4i 12j 4k
3 2

( 1, 1,2)

Therefore, the normal vector to the given surface is

n
4i 12j 4k
( 1, 1,2)

n 16 144 16 176 4 11

1
i 3j k
11

Ex. 3: Find the angle between the normals to the surface xy z at the points (1, 4, 2)
and (-3,-3, 3).
2

x, y, z xy z 2

Soln: Let

be the surface

The normal to the surface is

=i (xy-z 2 )+j (xy-z 2 )+k


(xy-z 2 )
x
y
z
=yi xj 2 zk

(1,4,2)


4i+j-4k

( 3, 3,3)


3i-3j+6k

Are normals to the surface at (1, 4, 2) and (-3,-3,3).


Let be the angle between the normals

cos =

n1 . n2

n1 n2

6k

4i j-4k . -3i-3j
16 1 16

9 9 36

12 3 24
33 54
39

33 54

cos 1

39

33 54

DEPT OF MATHS, SJBIT

Page 90

ENGINEERING MATHEMATICS-I

15MAT11

Ex.4: Find the directional derivative of x 2 yz 4 xz 2 at the point

(-1,-2,-1) in the direction of the vector 2i - j- 2k.

=x 2 yz + 4xz 2
2
(x 2 yz+4xz 2 )j+ (x 2 yz+4xz 2 )k
(x yz+4xz 2 )i+
x
y
z
(2 xyz 4 z 2 )i ( x 2 z )j+(x 2 y+8xz )k

Soln: Given

(1,2,1) 8i - j-10k

The directional derivative of at the point (1,-2,-1) in the direction of vector


is
2i - j- 2k.

2i - j- 2k

(8i - j-10k).

4 1 4

1
37
16 1 20

3
3

Ex.5: Find the directional derivative of xy 2 yz 3 at the point (1,-2,-1) in the


direction of the normal to the surface x log z y 2 4at (1, 2,1).
xy 2 yz3

y 2i 2 xy z 3 j 3 yz 2 k

Soln: Given

(1, 2, 1) 4i 5 j 5k

Let x log z y 2
1
Therefore the normal vector to the surface x log z y 2 4 is 1
x
=logzi -+(-2y)j k
1
z

1
and n
log(1)i (2 2)j k 4j k
1 ( 1, 2,1)
1

n 16 1 17

n
n

1
4j k
17

Therefore, the directional derivative of the surface xy 2 yz 3 at (1,-2,-1)


in the direction normal to the surface
2
x log z y 4at (1, 2,1). is
1
1
25
.n 4i 5 j 5k .
4 j k
.
20 5
17
17
17

DEPT OF MATHS, SJBIT

Page 91

ENGINEERING MATHEMATICS-I

15MAT11

Ex.6: Find the equation of tangent plane and normal to line to the surface x log
z y 2 4at the point (1, 2,1).

Soln: Let x log z y 2 .

=logz
x
x

( 1,2,1)

= 2y
y
y

( 1,2,1)

Therefore

=
z
z
z

=0
=4
= 1

( 1,2,1)

Thertefore, equation of the tangent plane is





(x x )
+( y y )
+( z z )
1 x
1 y
1 z
p
p

0
p

Hence ( x , y , z ) ( 1, 2,1), then


1 1 1
( x 1)0 ( y 2)( 4) ( z 1)( 1) 0
4 y 8 z 1 0
4y z 9 0
And the equation of the normal line is

x x1 y y1 z z1 x 1 y 2 z 1

x
z
y
Properties of divergence:


1.Prove that div A B div A div B


Or . A B . A . B

1. Pr oof: Let A A1i A2 j A3k, B B1i B2 j B3k


A B A1 B1 i A2 B2 j A3 B3 k


. A B A1 B1 A2 B2 A3 B3
y
z

A A A B B B
1 2 3 1 2 3
z x y
z
x y


. A B . A . B

DEPT OF MATHS, SJBIT

Page 92

ENGINEERING MATHEMATICS-I

15MAT11

2. Prove that

div A grad . A div A



Or . A . A . A

Pr oof: Let A A1i A2 j A3k

then , A A1i A2 j A3k


. A A1 A2 A3 by the property,
y
z

x
A
A
A

1 A1
2 A2
3 A3
x
x
y
y
z
z
A A A

1 2 3 A1
A2
A3
z
x
y
z
x y



. A A1i A2 j A3k . i
j
k
y
y



. A . A . A

Properties of curl :
1. Prove that curl:

A1 B1 curl A curl B

Or A1 B1 A B

Pr oof: Let A A1i A2 j A3 k,

Therefore, A B

A1 B1 i A2 B2

y
A2 B2

j A3 B3 k


A B

A1 B1
i

x
A1

B B1i B2 j B3 k

z
A3 B3

j
k
i

y z
x
A2
A3
B1

j
k

y z
B2 B3

A1 B1 A B

DEPT OF MATHS, SJBIT

Page 93

ENGINEERING MATHEMATICS-I

15MAT11

2. If A is a vector function and is a scalar function then

curl A curl A grad A

Or A A A

Pr oof: Let A A1i A2 j A3 k

then , A A1i A2 j A3k

x
A1

y
A2

z
A3


A3 A3 i
z
y

A3
A2



A3

A2
i
y
z
z
y
A
A2


3
A2
i
A3
z
y
z
y
A
A2
3
z
y
i j
k

x
A1

A2
i A3
y
z

i j
k

y z
x
A2 A3
A1


y z
A2 A3

A A A

Laplacian: The Laplacian operator 2 is defined by

2
2
2
2 2 2
x y z
2

Irrotational Vector (or Conservative Force Field): A vector field F is said to be

irrotational vector or conservative force field or curl free vector if F 0 or curl

F 0.

Scalar Potential: A vector field F which can be derived from the scalar field such

that F is called conservative force field and is called scalar potential.

DEPT OF MATHS, SJBIT

Page 94

ENGINEERING MATHEMATICS-I

15MAT11

Solenoidal Vector Function: A vector A is said to be solenoidal vector or divergence

free vector if div A . A 0.

Curl of a vector function: If A is any vector function differentiable at each point

(x,y,z) then curl of A is denoted by curl A or A and it is defined by


curl A A
x y z
A
A
A
1
2
3

A A A A
A
A
curl A A 3 2 i 3 1 j 2 1 k
x
y
z x
z
y

Hence, curl of a vector function is a vector.

Ex.1: If F grad x3 y y 3 z z 3 x x 2 y 2 z 2 then find div F at (1, 2,3).


Soln: Given

F grad x3 y y3 z z 3 x x 2 y 2 z 2

x3 y y 3 z z 3 x x 2 y 2 z 2

F 3x3 y z 3 2 xy 2 z 2 i x3 3 y 2 z 2 x 2 yz 2 j y 3 3z 2 x 2 x 2 y 2 z k

div F 6 xy 2 y 2 z 2 6 yz 2 x 2 z 2 6 zx 2 x 2 y 2

12 72 36 18 18 8 32.

1, 2,3

Ex.2 : If F =(x+y+1) i j ( x y)k, then show that F .curl F =0.

Soln: Given F =(x+y+1) i j ( x y)k,


j
i
k

curl F F
x
y
z
x y 1 1 ( x y)

DEPT OF MATHS, SJBIT

Page 95

ENGINEERING MATHEMATICS-I

15MAT11

(1 0)i (1 0) j (0 1) k

curl F i j k

F .curl F

x y 1 i j ( x y)k . i j k

x y 1 1 ( x y )
0

Ex.3: If F (ax 3 y 4 z )i ( x 2 y 3z ) j (3x 2 y z )k is solenoidal, find ;a.


Soln: Given

F (ax 3 y 4 z )i ( x 2 y 3z ) j (3x 2 y z )k, then

. F (ax 3 y 4 z ) ( x 2 y 3z ) (3x 2 y z ) a 2 1
x
y
z

. F a 3

Since the vector field is solenoidal therefore . F 0, then


a 3 0 a 3
Ex.4: Find the constants a,b,c such that the vector

F ( x y az )i ( x cy 2 z )k (bx 2 y z ) j is irrotational.

Soln: Given F ( x y az )i ( x cy 2 z )k (bx 2 y z ) j

Since the vector field is irrotational, therefore F 0 .


curl F
x
x y az

bx 2 y z x cy 2 z

(c 1)i (1 a ) j (b 1)k
i.e., (c 1)i (1 a ) j (b 1)k 0
This is possible only when,
c-1=0, 1-a=0, b-1=0 a=1, b=1, c=1.

DEPT OF MATHS, SJBIT

Page 96

ENGINEERING MATHEMATICS-I
n2

15MAT11

r,
Ex.5: Prove that 1. r nr
Soln: We have by the relation x=rcos , y=r sin .
By definition
n

r n i
r n j
rn
x
y
z
r
r
nr n 1
i nr n 1
j nr n 1
x
y

r n

k
r
k
z

But r 2 x 2 y 2
2r

r
r
x
2x

lll ly
x
x r

r
y
,
y
r

r
z

z r

x
y
z
nr n 1 i nr n 1 j nr n 1 k
r
r
r
nr n 2 xi nr n 2 yj nr n 2 xzk

nr n 2 xi yj zk

nr n 2 r

Ex.6: Prove that 2 f (r ) f (r )

2
f (r ), wherer r 2 x 2 y 2 z 2.
r

Soln: Given r 2 x2 y 2 z 2 .
r
r x
r y
r z
2r
2x
similarly
,

x
x r
y r
z r
2
2

r
Let f ( x) 2

f ( x) f (r )
2
2
x
x
x
x
2 2 2 2
Where

2
2
2
x
x
y
z 2

x
xf (r )
f (r )
x
r
x r

1
r2

r

r x xf (r ) xf (r ) x

1
r2


r
x

r xf (r ) x 1. f (r ) xf (r ) r

1
x
x
r xf (r ) f (r )
f (r )
r
r

r 2

DEPT OF MATHS, SJBIT

Page 97

ENGINEERING MATHEMATICS-I
1

r2

15MAT11

x2

x 2 f (r ) rf (r )
f (r )
r

1 2
x2
x f (r ) r
f (r )

r 2

x2
1
x 2 f (r ) r
f (r )

r2

1 2
z2
f ( r )

x f ( r ) r

r 2

1
r2

y2
x 2 f (r ) r

f (r )

2
2
2

x 2 y 2 z 2 f (r ) 1 r x r y r z f (r )

r
r
r
r 2

1 2
1
1 2

r f (r )
3r
x y 2 z 2 f (r )

r2
r2
1 2
1
1 2

r f (r )
3r
r f (r )

r2
r

r2
1 2
1

r f (r )
3r r f (r )

r2
r2

1
r2

2 f ( r ) f ( r )

3
f ( r )
r

Ex.7: Find the constants a and b so that F axy z3 i 3x 2 z j bxz 2 y k

irrotational and find such that F .

Soln: Given F (axy z3 )i (3x 2 z ) j (bxz 2 y )k

Since F is irrotational i.e., F 0 .

x
axy z 3

y
3x 2 z

0
z
bxz 2 y

i.e., (1 1)i (bz 2 3 z 2 ) j (6 x ax) k 0


i.e., z 2 (b 3) j (6 a) 0
which holds good if any only if b-3=0 and 6-a=0 a =6 and b=3.
DEPT OF MATHS, SJBIT

Page 98

ENGINEERING MATHEMATICS-I

15MAT11

Also given that F


j k 6 xy z 3 i (3x 2 z ) j (3xz 2 y )k
i

y
z

6 xy z 3 3x 2 y xz 3 f ( y, z )
1
x

3x 2 z 3x 2 y yz f ( x, z )
2
y

3xz 2 y xz 3 yz f ( x, y )
2
z
Hence 3x 2 y xz 3 yz.

DEPT OF MATHS, SJBIT

Page 99

ENGINEERING MATHEMATICS-I

DEPT OF MATHS, SJBIT

15MAT11

Page 100

ENGINEERING MATHEMATICS-I

DEPT OF MATHS, SJBIT

15MAT11

Page 101

ENGINEERING MATHEMATICS-I

DEPT OF MATHS, SJBIT

15MAT11

Page 102

ENGINEERING MATHEMATICS-I

15MAT11

1.

2.

DEPT OF MATHS, SJBIT

Page 103

ENGINEERING MATHEMATICS-I

DEPT OF MATHS, SJBIT

15MAT11

Page 104

ENGINEERING MATHEMATICS-I

15MAT11

MODULE IV
INTEGRAL CALCULUS
CONTENTS:
Introduction.....106

Reduction formulae for the integrals of sinnx, cosnx, sinmx


cosnx..107
Evaluation of these integrals with standard limits problems..108

DIFFERENTIAL EQUATIONS
Solution of first order and first degree equations110
Exact equations...114
Orthogonal trajectories121

DEPT OF MATHS, SJBIT

Page 105

ENGINEERING MATHEMATICS-I

15MAT11

Reduction formula:
1. Reduction formula for

sin

/2
n

sin

x dx and

x dx, n is a positive integer.

Let

I n sin n x dx
=

sin

n 1

x .sin x dx u v dx ( say )

We have the rule of integration by parts,


u v dx u v dx v dx.u dx

I sin n 1 x( cos x) cos x n 1 sin n 2 x.cos x dx


n
sin n 1 x.cos x (n 1) sin n 2 x.cos 2 x dx
sin n 1 x.cos x (n 1) sin n 2 x(1 sin 2 x) dx
sin n 1 x.cos x (n 1) sin n 2 x dx (n 1) sin n x dx
i.e., I sin n 1 x.cos x (n 1) I
(n 1) I
n
n2
n
i.e., I 1 (n 1) sin n 1 x.cos x (n 1) I
n
n2
sin n 1 x.cos x n 1
n
I sin x dx

I
n
n
n n2
This is the required reduction formula.
2. . Reduction formula for

co s x
n

/2

and

cos

x dx,

Where n is a positive integer.


I n cos n x dx
Let
= cos n 1 x .cos x dx

DEPT OF MATHS, SJBIT

Page 106

ENGINEERING MATHEMATICS-I

15MAT11

I cos n 1 x.sin x sin x n 1 cos n 2 x( sin x) dx


n
cos n 1 x.cos x (n 1) cos n 2 x.sin 2 x dx
cos n 1 x.sin x (n 1) cos n 2 x(1 cos 2 x) dx
cos n 1 x.sin x (n 1) cos n 2 x dx (n 1) cos n x dx
I cos n 1 x.sin x (n 1) I
(n 1) I
n
n2
n
i.e., I 1 (n 1) cos n 1 x.sin x (n 1) I
n
n2
cos n 1 x.sin x n 1
n
I cos x dx

I
n
n
n n2
/2

Next, let I
n

cos

xdx

/2

cos n 1 x.sin x
n 1

from (1), I
I
n
n
n n2

0
But cos / 2 0 sin 0.
\

n 1
Thus I
I
n
n n2

1. Reduction formula for

sin

/2
m

x cos x dx and

sin

cos n x dx where m and n

are positive integers.

x sin x cos x dx u v dx (say )

I m,n sin m x cos n xdx


=

sin

m 1

DEPT OF MATHS, SJBIT

Page 107

ENGINEERING MATHEMATICS-I

15MAT11

we have u v dx u v dx v dx.u dx
Here u dx sin x cos n dx
Put cos x t sin xdx dt
Hence

t n 1
cos n 1 x

n 1
n 1
n 1
cos x cos n 1 x
sin m 1 x
(m 1)sin m 2 x cos xdx

n 1
n 1

n
v dx t dt

Now I m,n

sin m 1 x cos n 1 x m 1

sin m 2 x cos n 2 xdx

n 1
n 1
m 1
n 1
sin x cos x m 1

sin m 2 x.cos n x.cos 2 xdx


n 1
n 1
sin m 1 x cos n 1 x m 1

sin m 2 x cos n x(1 sin 2 x)dx

n 1
n 1
m 1
n 1
sin x cos x m 1
m 1

sin m 2 x cos n xdx


sin m x cos n xdx

n 1
n 1
n 1
m 1
n 1
sin x cos x m 1
m 1
I m,n

I m 2, n
I m ,n
n 1
n 1
n 1
1
m 1
sin m 1 x cos n 1 x (m 1) I m 2, n
i.e., I m ,n 1

n 1 n 1
i.e.,

1
m 1
sin m 1 x cos n 1 x (m 1) I m 2, n
I m,n

n 1 n 1
sin m 1 x cos n 1 x m 1
m
n
I m ,n sin x cos xdx

I m 2, n ......(1)
mn
mn

PROBLEMS:

sin

1. Let I

x dx

f ( x) sin 4 x and 2a or a / 2
f (2a x) sin 4 ( x) sin 4 x f ( x)
2a

Thus by the property

I 2

sin
0

f ( x)dx 2 f ( x)dx we have,

/2

ie., f (2a x) f ( x)

3 1
x dx 2. . . by reduction formula.
4 2 2

Thus I=3 / 8

DEPT OF MATHS, SJBIT

Page 108

ENGINEERING MATHEMATICS-I

15MAT11

x sin

2. Let I

x dx

0
a

We have the property f ( x) dx f ( a x) dx


0

I ( x) sin 8 ( x)dx ( x) sin 8 xdx


0

sin 8 xdx x sin 8 xdx

I sin 8 xdx I
0

or 2 I .2

/2

sin

xdx

7 5 3 1
Hence I . . . . . , .
8 6 4 2 2
Thus by reduction formula
I=

35 2
256

3. Let I

x sin

x cos 4 x dx

( x) sin

( x) cos 4 ( x) dx, by a property.

( x ) sin 2 x cos 4 xdx


0

sin 2 x cos 4 x dx x sin 2 x cos 4 x dx

sin 2 x cos 4 x dx I
0

/2

2 I .2

sin 2 x cos 4 x dx

(1).(3).(1)
.
by reduction formula.
6 4 2 2
Thus I= 2 / 32
I .

DEPT OF MATHS, SJBIT

Page 109

ENGINEERING MATHEMATICS-I

15MAT11

2.

Introduction:
Many problems in all branches of science and engineering when analysed for
putting in a mathematical form assumes the form of a differential equation.
An engineer or an applied mathematician will be mostly interested in obtaining a solution
for the associated equation without bothering much on the rigorous aspects. Accordingly
the study of differential equations at various levels is focused on the methods of solving
the equations.

Preliminaries:
Ordinary Differential Equation (O.D.E)
If y = f (x) is an unknown function, an equation which involves atleast one derivative of
y, w.r.t. x is called an ordinary differential equation which in future will be simply
referred to as Differential Equation (D.E).
The order of D.E is the order of the highest derivative present in the equation and the
degree of the D.E. is the degree of the highest order derivative after clearing the
fractional powers.
Finding y as a function of x explicitly [y = f (x)] or a relationship in x and y satisfying the
D.E. [f (x, y)= c] constitutes the solution of the D.E.
Observe the following equations along with their order and degree.
DEPT OF MATHS, SJBIT

Page 110

ENGINEERING MATHEMATICS-I
1

dy
2x
dx

15MAT11
[order = 1, degree = 1]

dy
dy
2 3 2 0
dx
dx

[order = 1, degree = 2]

General solution and particular solution:


A solution of a D.E. is a relation between the dependent and independent
variables satisfying the given equation identically.
The general solution will involve arbitrary constants equal to the order of the D.E.
If the arbitrary constants present in the solution are evaluated by using a set of given
conditions then the solution so obtained is called a particular solution. In many physical
problems these conditions can be formulated from the problem itself.
Note : Basic integration and integration methods are essential prerequisites for this
chapter.

Solution of differential equations of first order and first degree


Recollecting the definition of the order and the degree of a D.E., a first order and
first degree equation will be the form
dy
f ( x, y ) or M(x,y)dx+N(x,y)dy=0
dx

We discuss mainly classified four types of differential equations of first order and first
degree. They as are as follows:

Variables separable equations

Homogenous equations

Exact equations

Linear equations

Variables separable Equations:


If the given D.E. can be put in the form such that the coefficient of dx is a
function of the variable x only and the coefficient of dy is a function of y only then the
given equation is said to be in the separable form.
The modified form of such an equation will be,
P (x) dx + Q (y) dy = 0
DEPT OF MATHS, SJBIT

Page 111

ENGINEERING MATHEMATICS-I

15MAT11

This is the general solution of the equation.


dy
y x2
Solve

xe
Example 1:
given that y(0)=0
dx
2
2
dy
dy
xe y x or
xe y e x
Soln: dx
dx

put - x 2 t 2 xdx dt or xdx dt


Hence we have, e

y et dt c

dy
x2

xe
dx by separating theVariables
ey

i.e.

e y dy xe x dx 0
2

i.e e y xe x dx c
2

The general solution becomes

e y

i.e.

et
c
2

e x
or
e y c is the general solution.
2
Now we consider y (0) =0 That is y =0 when x =0,
2

1
1
1 c or c =
2
2
Now the general solution becomes

e x
1
e y
2
2
2

This is the required solution.

DEPT OF MATHS, SJBIT

Page 112

ENGINEERING MATHEMATICS-I
Example 2 solve : xy

15MAT11

dy
1 x y xy
dx

dy
1 x y xy
dx
dy
i.e., xy
(1 x) y (1 x )
dx
dy
i.e., xy
(1 x)(1 y )
dx
ydy 1 x
or

dx by separating the variables.


1 y
x
y
1 x

dy
dx c
1 y
x
(1 y ) 1
1
or
dy dx 1dx c
1 y
x
1
ie., 1dy
dy log x x c
1 y
i.e y log y log x x c

xy

or y x log yx c is the required solution

Example 4 :

Solve : y x

dy
dy
y2
dx
dx

>> Rearranging the given equation we have,


dy
y y 2 x 1
dx
dx
dy
or

x 1
y y2

i.e log x 1

dy
(1)
y 1 y

We have to employ the method of partial fractions for the second term of the above.
Let

1
A
B

y 1 y y y 1

1 A y 1 By

Put y=0,1 A=-1 and B=1

DEPT OF MATHS, SJBIT

Page 113

ENGINEERING MATHEMATICS-I

15MAT11

dy
dy
dy

y 1 y
y
1 y
dy

y 1 y log y log 1 y log

1 y
y

Using this result in (1) we get,


1 y
log x 1 log
c
y
x 11 y
or log
log k
y

x 11 y ky is the required solution.

Example 5 :
Solve:
dy
tan y
cos( x y) cos( x y )
dx
The given equation on expanding terms in the R.H.S. becomes

dy
cos x cos y sin x sin y cos x cos y sin x sin y
dx
dy
ie., tan y
2 cos x cos y
dx
tan y

or

tany
dy 2 cos x dx by separating the variables.*
cosy

tany. sec y dy 2 cos x dx c


sec y 2sin x c is the required solution.

Exact Differential Equations:


The differential equation M(x , y) dx + N(x , y)dy=0 to be an exact equation is
M N

y
x
Further the solution of the exact equation is given by

M dx N ( y) dy c
Where, in the first term we integrate M(x,y) w.r.t x keeping y fixed and N(y) indicate the
terms in N with out x
(not containing x)

4
2 2
3
3
2 2
4
Solve: 5x 3x y 2 xy dx 2 x y 3x y 5 y dy 0

1.

DEPT OF MATHS, SJBIT

Page 114

ENGINEERING MATHEMATICS-I

15MAT11

(Though it is evident that the equation is a homogeneous one, before solving by putting
y=vx we should check for exactness)
Let M 5 x 4 3x 2 y 2 2 xy 3and N 2 x 3 y 3x 2 y 2 5 y 4

M
N
6 x 2 y 6 xy 2 and
6 x 2 y 6 xy 2
y
x
M
N
Since

, the given equation is exact.


y
x
The solution is M dx N ( y ) dy c
ie.,

5x

3 x 2 y 2 2 xy 3 dx 5 y 4 dy c

Thus x 5 x 3 y 2 x 2 y 3 y 5 c, is the required solution.

2.

2
Solve: cos x tan y cos( x y) dx sin x sec y cos( x y) dy 0

Let M cos x tan y cos( x y ) : N sin x sec 2 y cos( x y )


M
N
cos x sec 2 y sin( x y );
cos x sec 2 y sin( x y )
y
x
M N
Since

, the given equation is exact.


y
x
The solution is M dx N ( y ) dy c

ie.,

cos x tan y cos( x y)dx 0 dy c

Thus sin x tan y sin( x y ) c, is the required solution.

3.

Solve:

dy y cos x sin y y

0
dx sin x x cos y x

>> The given equation is put in the form,


y cos x sin y y dx sin x x cos y x dy 0.

Let M y cos x sin y y and N sin x x cos y x


M
N
cos x cos y 1 and
cos x cos y 1
y
x
M N
Since

, the given equation is exact.


y
x
The solution is M dx N ( y ) dy c

ie.,

y cos x sin y y dx 0 dy c

Thus y sin x x sin y xy c, is the required solution.

DEPT OF MATHS, SJBIT

Page 115

ENGINEERING MATHEMATICS-I
4.

15MAT11

Solve: ye xy dx xe xy 2 y dy 0
Let M ye xy , N xe xy 2 y
M
N
ye xy x e xy ;
xe xy y e xy
y
x
M N
Since

, the equation is exact.


y
x
solution is given by M dx N ( y ) dy c
ie., ye xy dx 2 y dy c
ie., y

e xy
y2 c
y

Thus e xy +y 2 c, is the required solution.

5. Solve: y(1 1/ x) cos y dx x log x x sin y dy 0


Let M y (1 1/ x) cos y and N x log x x sin y

M
N
1 1/ x sin y and
1 1/ x sin y
y
x
M
N
Since

, the given equation is exact.


y
x
The solution is M dx N ( y ) dy c

ie.,

y(1 1/ x) cos y dx 0 dy c

Thus y ( x log x) x cos y c, is the required solution.

Equations reducible to the exact form:


Integrating factor: Type-1:
Suppose that, for the equation M dx + N dy =0
M N

, then we take their difference.


y
x
M
N
should be close to the expression of M or N.

y
x
1 M N
1 M N
If it is so, then we compute

or

M y
x
N y
x

The difference

If

1 M N
1 M N

f ( x) or

g ( y)
N y
x
M y
x

f ( x ) dx
g ( y ) dy
or e
is an integrating factor.
Then e

DEPT OF MATHS, SJBIT

Page 116

ENGINEERING MATHEMATICS-I

15MAT11

The following basic results will be useful :


log x
n log x
(i) e
x (ii) e
xn
1.

2
Solve: 4 xy 3 y x dx x x 2 y dy 0

Let M 4 xy 3 y 2 x and N x x 2 y x 2 2 xy
M
N
4 x 6 y and
2 x 2 y. The equation is not exact
y
x
M
N
Consider

2 x 4 y 2( x 2 y )....close to N .
y
x

1 M N 2( x 2 y ) 2

f ( x)

N y
x x( x 2 y ) x
f ( x)dx
Hence e
is an integrating factor.
Now

f ( x)dx
ie., e
e

2
dx
2log x log( x 2 )
x
e
e
x2

Multiplying the given equation by x 2 we now have,


M 4 x3 y 3x 2 y 2 x 3 and N x 4 2 x 3 y
M
N
4 x3 6 x 2 y and
4 x3 6 x 2 y
y
x
Solution of the exact equation is M dx N ( y) dy c
ie.,

4 x y 3x
3

y 2 x 3 dx 0 dy c

Thus x 4 y x 3 y 2

x4
c, is the required solution.
4

Solve: y(2x-y+1)dx+x(3x-4y+3)dy=0
Let M y 2 x y 1 and N x 3x 4 y 3
2.

ie., M 2 xy y 2 y and N 3x 2 4 xy 3x
M
N
2 x 2 y 1,
6x 4 y 3
y
x
M N

4 x 2 y 2 2(2 x y 1)....near to M .
y
x
1 M N 2(2 x y 1)
2

g ( y)

M y
x y (2 x y 1)
y
2
dy
g ( y )dy
y
2 log y log( y 2 )
Hence I .F e
e
e
e
y2
Now

DEPT OF MATHS, SJBIT

Page 117

ENGINEERING MATHEMATICS-I

15MAT11

Multiplying the given equation by y 2 we now have,

M 2 xy 3 y 4 y 3 and N 3x 2 y 2 4 xy 3 3xy 2
M
N
6 xy 2 4 y 3 3 y 2 and
6 xy 2 4 y 3 3 y 2
y
x
The Solution is M dx N ( y ) dy c
ie.,

2 xy

y 4 y 3 dx 0 dy c

Thus x 2 y 3 xy 4 xy 3 c, is the required solution.

Integrating Factor: Type-2:


If the given equation M dx +N dy=0 is of the form
yf (xy) dx+xg(xy)dy=0
1
then
is an integrating factor provided Mx Ny 0
Mx Ny

2 2
2 2
Solve: y 1 xy x y dx x 1 xy x y dy 0

1.

>> The equation is of the form yf(xy)dx+xg(xy)dy=0 where,


M yf ( xy ) y xy 2 x 2 y3 and
N xg ( xy ) x x 2 y x3 y 2

Now Mx Ny xy x 2 y 2 x3 y3 xy x 2 y 2 x3 y3 2 x 2 y 2

1
1

is the I .F .
2
Mx Ny 2 x y 2

2 2
Multiplying the given equation with 1/ 2x y it becomes an exact equation where we now have,

1
1 y
1
1 x

and N

2
2
2
x
2
2
y
2
2x y
2 xy

The solution is given by M dx N ( y )dy c


1
1 y
1

2 x2 y 2 x 2 dx 2 y dy c

1
1
xy 1
ie.,
log x log y c
2 xy 2
2 2
ie.,

2.

Solve: y xy 2 x y
2

dx x xy x y dy 0
2

>> The equation is of the form yf(xy)dx+xg(xy)dy=0 where,


DEPT OF MATHS, SJBIT

Page 118

ENGINEERING MATHEMATICS-I

15MAT11

M xy 2 2 x 2 y3 and
N x 2 y x3 y 2
Now Mx Ny x 2 y 2 2 x3 y3 x 2 y 2 x3 y3 3x3 y3
3 3
Thus 1/ 3x y is the I.F. Multiplying the given equation by this I.F we have an exact equation

where we now have,


1
1
1
1
M

and N

2
2
3y
3x y 3x
3xy
The solution is M dx N ( y ) dy c
1
2
1

3x2 y 3x dx 3 y dy c

1
2
1
ie.,
log x log y c
3 xy 3
3
ie.,

Integrating factor: Type-3:


If the given equation Mdx+Ndy =0 is of the form

xk1 y k2 (c1 ydx c2 x dy) x k3 y k4 (c3 ydx c4 x dy) 0


Where ki and ci (i=1 to 4) are constants then x a y b is an integrating factor. The constants a and
b are determined such that the condition for an exact equation is satisfied.

1.

Solve: x(4y dx+2x dy)+ y 3 (3y dx+5x dy) = 0

>> We have (4 xy 3 y 4 )dx (2 x2 5xy3 )dy 0


Multiplying the equation by x a y b we have,

4(b 1) 2(a 2) and 3(b 4) 5(a 1)


ie., a 2b and 5a 3b 7
By solving we get a=2and b=1
We now have, M 4 x3 y 2 3x2 y5 and N 2 x4 y 5x3 y 4
The solution is Mdx N ( y)dy c

DEPT OF MATHS, SJBIT

Page 119

ENGINEERING MATHEMATICS-I

15MAT11

M 4 x a 1 yb 1 3x a yb 4 and
N 2 x a 2 yb 5 x a 1 yb 3
M
4(b 1) x a 1 yb 3(b 4) x a yb 3
y
N
2(a 2) x a 1 yb 5(a 1) x a yb 3
x
M N
We have to find a and b such that

y
x

ie.,

3 2
2 5
4 x y 3x y dx 0 dy c

Thus x 4 y 2 3x 2 y5 =c, is the required solution.


2.

Solve: ( y 2 2 x2 y)dx (2 x3 xy) dy 0

>>Multiplying the given equation by x a y b we have,

M x a yb 2 2 x a 2 yb 1 and
N 2 x a 3 yb x a 1 yb 1
M
(b 2) x a yb 1 2(b 1) x a 2 yb
y
N
2(a 3) x a 2 yb (a 1) x a yb 1
x
M N
Let us find a and b such that

y
x
(b 2) (a 1) and 2(b 1) 2(a 3)
ie., a b 3 and a b 2
By solving we get a = -5/2 and b=-1/2.

M x 5 / 2 y3 / 2 2 x 1/ 2 y1/ 2 and
We now have,
N 2 x1/ 2 y 1/ 2 x 3 / 2 y1/ 2
The solution is

Mdx N ( y)dy c

DEPT OF MATHS, SJBIT

Page 120

ENGINEERING MATHEMATICS-I

5 / 2 y3 / 2 2 x 1/ 2 y1/ 2 dx 0 dy c
x

ie.,
ie.,

15MAT11

x 3 / 2 3 / 2
x1/ 2 1/ 2
y
2
y
c
3 / 2
1/ 2
2 3 / 2 3 / 2
ie.,
x
y
4 x1/ 2 y1/ 2 c
3

Thus 6 xy y3 / x3 = k , is the required solution, where k=3c/2


Type-4 Exactness by inspection:
1. Solve : 1 y tan( xy) dx x tan( xy) dy 0
>> The given equation can be put in the form
dx tan ( xy ) y dx x dy 0

ie., dx tan( xy )d ( xy ) 0
Integrating we get, x logsec( xy) c , being the required solution.

2.

Solve:

y dx x dy
( x dx y dy ) 0
y2

>> The given equation is equivalent to the form,


x
d x dx y dy 0
y

x x2 y 2
c, on integration.
y 2 2
x 1
Thus x 2 y 2 c, is the required solution.
y 2

Orthogonal trajectories
Definition: If two family of curves are such that every member of one family intersect
every member of the other family at right angles then they are said to be
orthogonal trajectories each other

DEPT OF MATHS, SJBIT

Page 121

ENGINEERING MATHEMATICS-I

DEPT OF MATHS, SJBIT

15MAT11

Page 122

ENGINEERING MATHEMATICS-I

15MAT11

1.

2.

DEPT OF MATHS, SJBIT

Page 123

ENGINEERING MATHEMATICS-I

15MAT11

3) Find the orthogonal trajectories of the family of curves

x2
y2

1 ' ' being the parameter .


a 2 b2
Soln: we have

(July 2015)

x2
y2

1 ..........(1)
a 2 b2

Differentiating the (1) equation we get,


2x
2 y dy
2
0
2
a
b dx
x
y dy
i.e
2
........(2)
2
a
b dx
x2
y2

a2
b2
x2 a2
y2

.............(3)
a2
b2
Now , dividing (2) by(3) we get
x
y dy
2
2
2
x a
y dx
x
1 dy
2

2
x a
y dx
dy
dx
Now let us replace
by
dx
dy
Also from(1)

x
1 dx

2
x a
y dy
2

x2 a2
dx by separating the variables
x
dx
ydy xdx a 2 c
x
2
2
y
x
i.e

a 2 log x c is the required orthogonal trajectories


2
2
or ydy

4) Find the orthogonal trajectory of the cardiods r a 1 cos , using the


differential equation method .
Jan 2015 ,Jan2014)
Soln:

DEPT OF MATHS, SJBIT

Page 124

ENGINEERING MATHEMATICS-I
Differentiating the given equation w.r.t ,we get

15MAT11
dr
a sin .Substituting for a in the given equation,we get
d

1 cos dr
r
.........DE of given equation

sin d
dr
d
1 cos 2 d
Changing
to r 2
r
r

d
dr
dr
sin

dr
cos ec cot d 0........DE of orthogonal trajectories
r
solving this equation,we get

log r log cos ec cot log sin log c


r

cos ec cot c r 1 cos c

sin
sin 2
r c 1 cos , this is requried orthogonal trajectories.

DEPT OF MATHS, SJBIT

Page 125

ENGINEERING MATHEMATICS-I

15MAT11

MODULE V
LINEAR ALGEBRA
CONTENTS:
Elementary transformation127
Reduction of the given matrix to echelon and normal forms..128
Solution of a system of non-homogeneous equations by
Gauss Elimination method.136
Gauss Jordan method 141
Gauss Seidel method. ..... 143
Linear transformations..148
Reduction to diagonal form, quadratic forms.157
Reduction of quadratic form into canonical form..159
Rayleighs power method to find the largest Eigen value and
the corresponding Eigen vector163

DEPT OF MATHS, SJBIT

Page 126

ENGINEERING MATHEMATICS-I

15MAT11

Definition: A system of mn nos. arranged in a rectangular formation along m-rows &


n-columns & bounded by the brackets

or is called as m by n matrix or mxn matrix

Matrix is denoted by a single capital letters A,B,C etc.

a11 a12 .......a1n


A a21 a22 .......a2 n
am1 am 2 ......amn
mn

Elementary operations on Matrices:


The following 3 operations are said to be elementary operations
1. Interchange of any two rows or columns.
2. Multiplication of each element of a row or column by a non-Zero scalar or constant.
3. Addition of a scalar multiple of one row or column to anpother row or column.

a1 a2 a3

A b1 b2 b3
c1 c2 c3

R3 kR1 R3

a1 a2 a3

B b1 b2 b3
(ka1 c1 ) (ka2 c1 ) (ka3 c3 )

If a matrix A gets transferred into another matrix B by any of these transformations then
A is said to be equivalent to B written as A B.
Echelon form or Row reduced Echelon form.
A matrix A of order mxn is said to be in a row reduced echelon form if
1. The leading element (the first non-Zero entry) of each row is unity.
2. All the entries below this leading entry is Zero.
3. The no of Zeros appearing before the leading entry in each row is greater than that
appears in its previous row.
4. The Zero rows must appear below the non-zero rows.

1 2 3 2
A 0 0 1 2
0 0 0 1

1 2 3 2 1 4 2
0 1 2 9 0 1 0

0 0 0 0 0 0 0

Normal form of a matrix


The given matrix A is reduced to an echelon form first by applying a series of elementary
row transformations.
Later column transformations row performed to reduce the matrix to one of the
following four forms, called the normal form of A.

i) Ir

Ir
ii ) Ir , o iii )
o

DEPT OF MATHS, SJBIT

Ir o
iv)
where Ir is the identity matrix of order r.
o o

Page 127

ENGINEERING MATHEMATICS-I
1 0 0
0 1 0

0 0 1

I 3 I 3 , 0

1 0
1 0 0 0

0 1 0 0 0 1

0 0
0 0 1 0

0 0
I 2 I3 0
0 0 0

15MAT11

1
0

0
1
0
0

0 0
0 0
1 0

0 0

The number of non-zero rows in echelon or normal form. Its in


denoted by f(A)
1. Reduce the matrix to the row reduced echelon form

Rank of a matrix:

2 2 3
1
0
0 1 2

A
1 3 2 1

4 1 3
2
Sol : R2 R2 2 R1 , R3 R3 R1

R4 R4 2 R1

2 2 3
2 2
1
1
0

1 8 1
0
1 8
A
A
0 1 0 2
0
0 8

0 3 3
0 3
0
0
R3 R3 / 8, R4 R4
R4 R4 R3

3
1
3

1
0

A
0

2 2

2 2

0
0

1
3
0
8 1

A 0

3
1

8
0
1 3

0 1
0

3
8
11
8
3

8
R4
11
2 2 3
1
0
1 8 1

A
0
0 1 3
8

0
0
0 1

R4

DEPT OF MATHS, SJBIT

Page 128

ENGINEERING MATHEMATICS-I

15MAT11

1 3 1 2
2) 0 1 2 3 find rank of a matrix
3 4 1 2
Sol : R3 R3 3R1 , R3 R3 13R2
2
1 3 1 2
1 3 1

A 0 1
2 3 A 0 1
2
3
0 13 2 8
0 0 28 34
( A) 3

8 2 1 6
3) 2 1 0 1 find the rank
5 1 1 4
Sol : R2 4 R2 R1 , R3 8 R3 3R1
2
1 6
8
0
2 1 2

A
0 6
5
6

3 2
0 2
R4 R4 R3
8
0
A
0

2
2
0
0

8
0
A
0

R4 R4 R2
2
2
0
0

6
1 2
2
0

2 0
1

6
1 2
2
0

0
0
1

( A) 3

DEPT OF MATHS, SJBIT

Page 129

ENGINEERING MATHEMATICS-I

15MAT11

3 2
1

4) Using the elementary transformation reduce the matrix A 2 1


4
1 11 14
echelon form
Sol : R2 R2 2 R1 , R3 R3 R1
3 2
1

A 2 1
4
1 11 14

3 2

1
0 7
8

0 14 16
R
R3 R3 2 R2 , R2 2
7
2 2
1
2 2
1

A 0
8
0 7
8
1

0 0
0 0
0

to

5) Applying elementary transformations reduce the following matrix to the normal form &

3 2 5 7 12

hence find rank of matrix given 1 1 2 3 5

3 3 6 9 15

Sol : R1 R2
3 2 5 7 12
A 1 1 2 3 5
3 3 6 9 15
R2 R2 3R1 ,

1 1 2 3 5
3 2 5 7 12

3 3 6 9 15
R3 R3 3R1

1 1 2 3 5
A 0 1 1 2 3
0 0 0 0 0

This echelon form, now we have to perform column trans to reduce to the normal form.

DEPT OF MATHS, SJBIT

Page 130

ENGINEERING MATHEMATICS-I
c2 c2 c1 ,

15MAT11

c3 c3 2c1 c4 c4 3c1 c5 c5 5c1

1 0 0 0 0
A 0 1 1 2 3
0 0 0 0 0
c3 c3 c2 ,

c4 c4 2c2 c3 c5 3c2

1 0 0 0 0
A 0 1 0 0 0
0 0 0 0 0
I 0
A 2
( A) 2
0 0
6)

R2 R2

1
A 0
0

0 0 0 0
1 0 0 0
0 0 0 0

By performing elementary row & column transformations, reduce the following matrix to

2 4 3 1
0 2 1 4
the normal form
0 1 1 3

4 7 4 4

0
2
1

Sol : R2 R2
2 4 3 1
0 2 1 4
A
0 1 1 3

4 7 4 4

R2 R2 2 R1 ,

1 4

1 2

2 4 3 1

0
1 1 3

5
4 7 4 4

0
2
1

R4 R4 4 R2

1 2 1 4 2
1
0 0 1

2
9 4
A

0 1 1 3 1
0

0 1 0 12 3
0
c2 c2 2c1 ,

2
2
1

R2 R3

2 1 4 2
1 1 3 1

0 1 9 4

1 0 12 3

c3 c3 c1 c4 c4 4c1 ,

c5 c5 2c1

1 0 0 0 0
0 1 1 3 1

0 0 1 9 4

0 0 0 0 0

DEPT OF MATHS, SJBIT

Page 131

ENGINEERING MATHEMATICS-I

c3 c3 c2 ,
1 0
0 1

0 0

0 0
I
A 3
0

c4 c4 3c2 ,

1 0

0 1
0 0 1

0 0
1 9 4

0 0 0
0 0
0

0
0

0 0

15MAT11

c5 c5 c2
0 0 0
0 0 0
1 0 0
0 0 0

c4 c4 9c3 c5 c5 4c3

( A) 3
1 1
1 1
7) Re ducing the matrix A
3 1

2 2

R2 R2 R1 ,
1 1
1 1
A
3 1

2 2

16
2 5
int o normal form and find the rank
1 8

3 7
R3 R3 3R1 R4 R4 2 R1

1 6 1 1 1 6
2 5 1 2 1 1

1 8 0 2 2 10

3 7 0 4 1 5

R3 R3 R2 ,

R4 R4 2 R2 R3

1 1 1 6 1 1 1 6
1 2 1 1 1 2 1 1

A
0 0 3 9 0 0 1 3


0 0 1 3 0 0 1 3

DEPT OF MATHS, SJBIT

1
R3
3

Page 132

ENGINEERING MATHEMATICS-I
R2 R2
1
0
A
0

0
c
c3 2

15MAT11

c3 2c3 2c2 c4 2c4 c2

0 1
2 1 1 0

0
1 3 0

0 0 0 0
0 0

0 0 0
2 1 1
0 1 3

0 0 0

c c4 3c3
2 4
1 0 0 0
0 1 0 0

0 0 1 0

0 0 0 0
I 0
3 ( A) 3.
0 0

1 2 2 3
2 5 4 6

8) Find the rank of the matrix


1 3 2 2

2 4 1 6
Sol : R2 R2 2 R1 , R3 R3 R1 R4 R4 2 R1

1 2 2 3
2 5 4 6
A
1 3 2 2

2 4 1 6
R3 R3 R2 ,
1 2 2 3
0 1 0 0
A
0 0 0 1

0 0 3 0

c2 c2 2c1 ,
1
0

1 2 2
0 1 0

0 1 0

0 0 3
1
R3 R4 R3 R3
3
1 2 2 3
0 1 0 0

0 0 1 0

0 0 0 1

3
0
1
0

c3 c3 2c1 c4 c4 3c1

0 0 0
1 0 0
( A) 4
0 1 0

0 0 1

DEPT OF MATHS, SJBIT

Page 133

ENGINEERING MATHEMATICS-I

15MAT11

9) Find the rank of the matrix by reducing to the normal form

1 1 1 1
1 2 3 4

2 3 5 5

1)
3 4 5 8
Sol : R2 R2 R1 , R3 R3 2 R1 R4 R4 3R1
2)

1 1 1 1 1 1 1 1
1 2 3 4 0 1 2 5

A
2 3 5 5 0 1 3 7

3 4 5 8 0 7 8 5
R3 R3 R2 , R4 R4 7 R2 R4 R4 6 R3
1 1 1 1
1
0 1 2 5 0

A
0 0 1 2 0


0 0 6 30 0
c2 c2 c1 , c3 c3 c1
1
0

1
1 2 5 0

0 1 2 0

0 0 18 0

1 2 5
0 1 2

0 0 18
c4 c4 c1 c3 c3 2c2 c4 c4 5c2
1 1 1

1 0 0
0 1 2

0 0 18
1
c4 c4 2c3 c4 c4
18
1 0 0 0
1 0 0 0
0 1 2 0 0 1 0 0

0 0 1 0 0 0 1 0

0 0 0 18 0 0 0 1
( A) 4.
0 0 0

0 0 0

DEPT OF MATHS, SJBIT

Page 134

ENGINEERING MATHEMATICS-I

15MAT11

1 1 1 1

10) Find the value of K such that the following matrix A = 1 2 4 k may have rank equal
1 4 10 k 2

to a) 3 b) 2.
3)

Sol : R2 R2 R1

R3 R3 R1

1 1 1 1 1 1 1 1

A 1 2 4 k 0 1 3 k 1
1 4 10 k 2 0 3 9 k 2 1


R3 R3 3R2

1 1 1 1

0 1 3 k 1

2
0 0 0 k 3k 2
a) Rank of A can be 3 if the equivalent form of A has 3 non-Zero rows.

This is possible if k 2 3k 2 0
i.e, (k 1)(k 2) 0
( A) 3 if k 1 & k 2
4) b) Rank of A can be 2 if the equivalent form of A has 2 non-zero rows.
5)

This is possible if k 2 3k 2 0

i.e, (k 1)(k 2) 0 k 1 or k 2
( A) 2 if k 1 & k 2
6)
a11a12 .........a1n : b1
A : B a21a22 .........a2 n : b2
am1am 2 .........amn : bm
The given sys of equation is consistent & will have unique soln.
Let us convert A : B into a set of equation as follows

x+ y+ z = 6
-2y + z = 7
-3z = -9
z= 3

-2y+3=-1
-2y=-4

x+y+z=6

y=2

x=1

x=6-2-3=1

x=1, y=2, z=3is the unique soln.

DEPT OF MATHS, SJBIT

Page 135

ENGINEERING MATHEMATICS-I

15MAT11

7) Solve the system of equations: x+2y=3z=0


2x+3y+z=0
4x+5y+4z=0
x+y-2z=0
1
2
3
:0

2 3 1 :0

Sol : A : B
4 5 4 :0

1 1 2 : 0
R2 R2 2 R1
R3 R3 4 R1
R4 R4 R1 R3 R3 3R2
1
0

3 :0
1 5 :0

3 8:0

1 5: 0
( A) 3
2

R4 R4 R2

1 2 3 :0
0 1 5 :0

0 0 7:0

0 0 0 : 0

A : B 3 n 3
Hence the system is consistent & will have trivial soln x=0 y=0 z=0
8) Does the following system of homogenous equations possess a non-trivial
solutions? If so find them
x1 x2 x3 x4 0

x1 x2 2 x3 x4 0
3 x1 x2 x4 0
1 1 1 1 :0
A : B 1 1 2 1 :0
3 1 0 1 :0

R2 R2 R1

R3 R3 3R1

R3 R2

1 1 1 1 :0 1 1 1 1 :0
0 2 3 2 :0 0 2 3 2 :0
0 2 3 2 :0 0 0 0 0 :0

Gauss elimination method:


The simplest method of solving systems of the form (1) of section 5.2 is the
elimination method.
The Working Rule for the method is as given below.

DEPT OF MATHS, SJBIT

Page 136

ENGINEERING MATHEMATICS-I

15MAT11

Working rule:
Step1: Reduce the augmented matrix (A:B) to the form where A is in echelon form or in
upper triangular form, by employing appropriate elementary row operations.

Step2: Write the linear equations associated with the reduce form obtained in Step 1. Let
the number of equations in this reduced system be equal to r, If r=n, then the reduced
system yields the unique solution the given system. If r<n, then n-r unknowns in the
reduce system can be chosen arbitrarily and the reduced system yields infinitely many
solutions of the given system.
1. : Solve the following system of linear equations by the Gauss elimination method
x1 x2 x3 4
2 x1 x2 x3 1
x1 x2 2 x3 2

1 2 1
Sol: For the given system, the coefficient matrix is A 2 1 1
1 1 2
1 1 1 : 4
And the augmented matrix is A : B 2 1 1: 1
1 1 2 : 2

We reduce this matrix A : B to the upper triangular form by using elementary operations
Using the row operation R2 R2 2R1 and R3 R3 R1 ,We get

1 1 1 : 4
A : B 0 1 3: 7
0 2 1 : 2

Now, Using the row operation R 3 R 3 2 R 2 in this, we get


1 1 1 : 4
A : B 0 1 3: 7
0 0 7 : 12
We note that A is now reduced to the upper triangular form. The linear equations which
correspond to this reduced form of A : B are

DEPT OF MATHS, SJBIT

Page 137

ENGINEERING MATHEMATICS-I

15MAT11

x1 x2 x3 4
x2 3x3 7
7 x3 12
From equation (iii), wew find that x3 12 / 7.
36 13

7
7
Substituting for x3and x2 found above in (i), we get
x2 7 3x3 7

13 12 3
= .
7 7 7
Thus, x1 =3/7, x2 =13/7, x3 =12/7 constitute the solutionof the given system.
x1 4 x2 x3 4

2.

Solve the following system of equations by Gausss elimination method:


4 x1 x2 x3 4
x1 4 x2 2 x3 4
3x1 2 x2 4 x3 6

4 1 1 : 4
Sol: The augmented matrix is A : B 1 4 2: 4
3 2 4 : 6
1 :4
4 1

A : B 0 15 / 4 9 / 4 : 3
0 10
2 : 6
Using R 2 R 2 (1/ 4)R1 , R 3 R 3 3R 2
4 1 1 : 4
A : B 0 5 3 : 4
0 5 1 : 3
Using R 2 (4 / 3)R 2 , R 3 (1/ 2)R 3

4 1 1 : 4
A : B 0 5 3 : 4
0 0 2 :1
Using R 3 R 3 R 2 ,
We note that A is now reduced to the upper triangular form. The equations that
correspond to (i) are

DEPT OF MATHS, SJBIT

Page 138

ENGINEERING MATHEMATICS-I

15MAT11

4 x1 x2 x3 4
5 x2 3x3 4
2 x3 1.
These yield .
1
1
1
1
x3 , x2 (4 3 x3 ) , x1 (4 x2 x3 ) 1.
2
5
2
4
Thus, x1 =1, x2 =1/2, x3 =-1/2 constitute the solution of the given system.

3. Solve the following system of equations by Gausss elimination method:


x 2y 2y 1

2x y z 2
3x 2 y 2 z 3
x z 0
1 2 2 :1
2 1 1: 2

Sol: The augmented matrix is A : B


3 2 2 : 3

0 1 1 : 0
1 2 2 :1
0 3 3: 0
A : B 0 4 4: 0

0 1 1 : 0
Using R 2 R 2 2R1 , R 3 R 3 3R1
1 2 2 :1
0 1 1: 0
A : B 0 1 1: 0

0 1 1 : 0
Using R 2 (1/ 3)R 2 , R 3 (1/ 4)R 3
1 2 2 :1
0 1 1: 0
A : B 0 0 0: 0

0 0 0 : 0
Using R 3 R 3 R 2 , R 4 R 4 R 2 ,
We note that A is now reduced to the echelon form. The system correspond to (i) is
x 2 y 2z 1
yz 0

DEPT OF MATHS, SJBIT

Page 139

ENGINEERING MATHEMATICS-I

15MAT11

These are two equations for three unknowns. Therefore, we can choose one of the unknown arbitrarily. Taking z=k, we get y=-k and x=1
Thus x=1,y=-k, z=k, Where k is arbitrary, is a solution of the given system.
5) Solve the following system of equations by Gausss elimination method:

5 x1 x2 x3 x4 4
x1 7 x2 x3 x4 12
x1 x2 6 x3 x4 5
x1 x2 x3 x4 6

Sol : Consider augmented matrix A : B by R1 R4


1
1
A : B 1

5
R2 R2 R1 ,

1 1 4 : 6
7 1 1 : 12
1 6 1 : 5

1 1 1 : 4
R3 R3 R1 , R4 R4 5R1

4
1 1 1
0 6 0 3
A : B 0 0 5 3

0 4 4 19
R4 R4 2 R2
4
1 1 1
0 2 0 1
A : B 0 0 5 3

0 0 4 21
R4 5 R4 4 R3

: 6
: 18

: 1

: 34

: 6
0 2 0
1 : 6
0 0 5 3 : 1

0 4 4 19 : 34

: 6
: 6
: 1

: 46

4
:
1 1 1
0 2 0 1 :
A : B 0 0 5 3 :

0 0 0 117 :
Hence we have x1 x2 x3 4 x4

6
6
1

234
6

2 x2 x4 6
5 x3 3 x4 1
117 x4 234
x4 2, x3 1, x2 2 and x1 1is the reqd . so ln .

DEPT OF MATHS, SJBIT

Page 140

ENGINEERING MATHEMATICS-I

15MAT11

Gauss -Jordan method:


This method can be regarded as the modification of Gauss elimination method.
This method aims in reducing the coefficient matrix A to a diagonal matrix.
Applying Gauss Jordan method solve 2 x 3 y z 5 , 4x 4 y 3z 3 , 2x 3 y 2z 2

1)

2 3 1 : 5

3 : 3
Soln: A : B 4 4

2 3 2 : 2
R2 R2 2 R1 , R3 R3 R1

2 3 1 : 5
A : B 0 2 1: 7
0 6 3 : 3
R1 2 R1 3R2 , R3 R3 R2

2 3 1 : 5
0 2
1:
7

0 2 1 : 1

4 0 5 : 11
7
A : B 0 2 1:
0 0 2 :
6
R1 R1 5R3 , R2 R2 R3

4 0 5 : 11
0 2
1:
7

0 0 1 :
3

4 0 0 : 4
A : B 0 2 0 : 4
0 0 2 : 6
Hence 4 x 4, 2 y 4, 2 z 6
x 1, y 2, z 3

2.

DEPT OF MATHS, SJBIT

Page 141

ENGINEERING MATHEMATICS-I

DEPT OF MATHS, SJBIT

15MAT11

Page 142

ENGINEERING MATHEMATICS-I

DEPT OF MATHS, SJBIT

15MAT11

Page 143

ENGINEERING MATHEMATICS-I

15MAT11

Problem 1

DEPT OF MATHS, SJBIT

Page 144

ENGINEERING MATHEMATICS-I

15MAT11

Problem 2

DEPT OF MATHS, SJBIT

Page 145

ENGINEERING MATHEMATICS-I

15MAT11

Problem 3

Soln:

DEPT OF MATHS, SJBIT

Page 146

ENGINEERING MATHEMATICS-I

DEPT OF MATHS, SJBIT

15MAT11

Page 147

ENGINEERING MATHEMATICS-I

15MAT11

LINEAR TRANSFORMATION:
A Linear transformation in two dimensions is given by
y1 a1 x1 a2 x2

y2 b1 x1 b2 x2
This can be represented in the matrix form as
y1 a1 a2 x1

) Y AX
y2 b1 b2 x2
Similarly a linear transformation in 3 dimensions along with its matrix form is as,
y1 a1 x1 a2 x2 a3 x3
y2 b1 x1 b2 x2 b3 x3
y3 c1 x1 c2 x2 c3 x3

A is called transformation matrix


If A 0 Then y=AX is called non-Singular transformation or regular transformation.
If A 0 Then y=AX is called Singular transformation
X= A1 y is called the inverse transformation.
Let z=By =B(AX)=(BA)X=CX Where C=BA Z=CX is called a composite linear
transformation .
1. Show that the transformation
y1 2 x1 x2 x3 ;
y2 x1 x2 2 x3
y3 x1 2x3 is regular. Write down the
inverse transformation.
Sol: The given transformation may be written as
Y=AX
y1 2 1 1 x1


y2 1 1 2 x2
y 1 0 2 x
3
3
2 1 1
A 1 1 2 2(2) 1( 2 2) 1(1) 4 4 1 1 0
1 0 2
A 0 A is a non-singular matrix

The transformation is regular


The inverse transformation is X= A1Y

DEPT OF MATHS, SJBIT

Page 148

ENGINEERING MATHEMATICS-I

15MAT11

2 2 1
adjA
A
4 5 3
A
1 1 1
X A1Y
1

x1 2 2 1
x 4 5 3
2

x3 1 1 1
x1 2 y1 2 y2 y3

y1

y2

y3

x2 4 y1 5 y2 3 y3

is the inverse transformation.

x3 y1 y2 y3
2. Prove that the following matrix is orthogonal
1
2
2
3
3
3

2
1
A 2
3
3
3
1
2
2
3
3
3
2
3
Sol:Consider AA I 2
3
1
3

3
2

3
1
3
2

2
3
1
3
2
3 3

2
2
1

3
3

3 1 0 0
2 0 1 0

0
0
1

2
3
1

1 2 a
3. If A=1/3 2 1 b is orthogonal. Find a,b,c & A1 A is orthogonal AA I
2 2 c
1 2 a
1 2 2
1 0 0
1
1

Sol : 2 1 b
2 1 2 0 1 0

3
3
2 2 c
a b c
0 0 1
1 4 a 2
2 2 ab
2 4 ac
9 0 0

2
4 2 bc 0 9 0
2 2 ab 4 1 b
2 4 ac
0 0 9
4 2 bc
4 4 c 2

5 a2 9

5 b2 9

8 c2 9

a2 4
a2

b2 4
b2

c2 1
c 1

DEPT OF MATHS, SJBIT

Page 149

ENGINEERING MATHEMATICS-I

15MAT11

1 2 2
1
AA I A A 2 1 2
3
2 2 1
1

4. Find the inverse transformation of the following linear transformation

y1 x1 2 x2 5 x3

y2 2 x1 4 x2 11x3
y3 x1 2 x3
Sol: Y=AX

x1 19 9
A Y x2 4
2
x3 2
1
x1 19 y1 9 y2 2 y3
1

x2 4 y1 2 y2 y3

2
1
0

y1

y2

y3

is the inverse transformation

x3 2 y1 y2
5. Represents each of the transformation y1 z1 2 z2 & x2 y1 4 y2 , y2 3z
by the use of matrix & find the composite transformation which express
x1 , x2 in terms of z1 , z2
Sol : x1 3 y1 2 y2

x2 y1 4 y2
x 3
x AY 1
x2 1
y1 z1 2 z2

2 y1
4 y2

y1 1 2 z1
y2 3 z1 y BZ

y2 3 0 z 2
X AY A( BZ ) ABZ AB Z
3 2 1 2 9 6

4 3 0 11 2

x1 9 6 z1
x 11 2 z
2
2
x1 9 z1 6 z2

AB 1

x2 11z1 2 z2 is the required composite transformation

6.` Given the linear transformation


DEPT OF MATHS, SJBIT

Page 150

ENGINEERING MATHEMATICS-I

15MAT11

y1 5 x1 3 x2 3 x3
y2 3 x1 2 x2 2 x3
y3 2 x1 x2 2 x3
Sol : z1 4 x1 2 x3
z2 x2 4 x3
z3 5 y3
Express

y1, y2, y3

int erms of z1 , z2 , z3
Given : Y=AX

Z BX X B 1Z

1
4
B 1 0

0
1
0

1
10
4
5
1
5

Y ( AB 1 ) Z
5
3 23
10 Z1
y1 4
3
23 z
y2 4 1
10 2
y
z3
3
1 1 1

Eigen values and Eigen vectors of a square matrix:


Definition: Let A be a given square matrix of order n. suppose I a non-zero Column
vector X of order n and real or complex no. Such that AX= x
Then X is called an Eigen vector of A.
is called the corresponding Eigen value of A.
Working Rule:
1. Given square matrix A write down the characteristic equation A I 0
2. Solve the characteristic equation for Eigen values 1 , 2 , 3 ,.......
3. To find Eigen vector, write down the matrix equation as
x1
A I X 0 where X x2
xn
4. We set 1 in the matrix equation & solve it for Eigen vector x1 . Similarly we
obtain Eigen vector x2 , x3 ..... for corresponding Eigen value 2 , 3 .....
DEPT OF MATHS, SJBIT

Page 151

ENGINEERING MATHEMATICS-I

15MAT11

1.

Find the Eigen values & corresponding eigen vector of the following matrix
3 8
A

2 7
3
8
Sol : A I
(3 )(7 ) 16
2
7

2 4 5
( 5) ( 1) 0
The roots of this equation are 1 =5 & 2 =-1.

These are the two Eigen value of the given

matrix A. Let X= x, y , then the matrix equation ( A I ) X 0


T

3 8 x 0

7 y 0
2
1 5

8 8 x 0
2 2 y 0


-8x+8y=0, -2x+2y=0. Both of these reduces to some equation x-y=0 =) x=y. If we choose
a
x=a, then y=a These, when 1 5, x1 is the solution of (1)
a

0 2 1 equation (1) becomes


-2 8 x 0
-2 8 y 0


For 2 x 8 y 0, x 4 y 0.Hence if we choose y=b, then x=4b
Ab
Thus when 2 1, x2 is the soln of (1)
b
2. Find the Eigen values & the Eigen vectors of the matrix
2 0 1
A 0 2 0
1 0 2
Sol: For the given matrix, the characteristic polynomial is
1
2 0

A I 0
2 0
1
0 2
(2 x)3 (2 )

( 3)(2 )( 1)
DEPT OF MATHS, SJBIT

Page 152

ENGINEERING MATHEMATICS-I

15MAT11

The characteristic equation of the given matrix is


(2 )( 3)( 1) 0
The roots are 1 1 2 2

x, y , z

& 3 3 these are the Eigen value of the given matrix.

X , then the matrix equation (A- I)x=0

1
2- 0

0
2 0

1
0
2
For 1 1

x 0
y 0 (1)

z 0

1 0 1 x 0
0 1 0 y 0


1 0 1 z 0
x z 0 & y 0
If we choose x=a, then z=-a; x1 a1 01 a

If a 1

x1 1 0 1

0
For 2 2 0
1

1
0
0

0
0
0

x 0
y 0

z 0

x 0, z 0. we take y=b; x 2 0 b 0

If b 2

Let

x 2 0 2 0

1 0 1 x 0
For 3 3 0 1 0 y 0
1 0 1 z 0
x z 0, y 0. we take x=c; then z=c x 3 c o c

If c 3

x 3 3 0 3

1 1
3. Find the matrix P which reduces the matrix A 1 5
3 1

3
1 to diagonal form
1

Hence find A4
3
1- 1

A I 1
5 1 0
3
1
1
1 2 2 3 3 6

DEPT OF MATHS, SJBIT

Page 153

ENGINEERING MATHEMATICS-I

15MAT11

1-
A I 1
3

1
3
5 1 0
1
1
1 2 2 3 3 6
3 1 3 x 0
Case(1): 1 7 1 y 0
3 1 3 z 0
3x y 3z 0
x 7y z 0
x
y
z

1 3 3 3 3 1
7 1

1 1

1 7

x
y z

20 0 20
Caseii ): 2 3
2 1 3 x 0
1 2 1 y 0


3 1 2 z 0
2 x y 3 z 0
x 2y z 0
3x y 2 z 0
x
y
z

2 1 1 1 1 2
1 2 3 2 3 1
x y z

5 5 5
Caseiii ): 3 6
5 1 3 x 0
1 1 1 y 0


3 1 5 z 0
5 x y 3 z 0
x yz 0
3x y 5 z 0
x
y
z

1 1 1 1 1 1
1 5
3 5 3 1
x MATHS,
y z SJBIT
DEPT OF

4 8 4

Page 154

ENGINEERING MATHEMATICS-I

15MAT11

x y z

4 8 4
x3 1 1 1
y3 0 1 2
1 1
z3 1

x1 x2
p x1 x2 x3 y1 y2
z1 z2

1
1
2 0 2

1
1 1
1

3
3 3

1 1 1
6 3 6
2 0 0
1
p AP 0
3 0 D
0
0 6
1 1 1
A PD p 0 1 2
1 1 1
4

251
405
235

405
891
405

16
0

0
81
0
0 1296

1
1
2 0 2

1 1 1
3
3 3

1 1 1
6 3 6

235
405
251

11
5) Diagonalizable the matrix A 7
10

4
2
4

7
5 and find A5
6

(A I) 0
11 4 7
7
2 5 0

10
4 6
3 3 3 2 0
2 3 2 0
x 0 1

DEPT OF MATHS, SJBIT

Page 155

ENGINEERING MATHEMATICS-I

15MAT11

Casei : 0
11
7

10

4
2
4

7
5
6

x 0
y 0

z 0

11x 4 y 7 z 0
7 x 2 y 5z 0
10 x 4 y 6 z 0
x
y
z

2 5
7 5
7 2
4 6
10 6 10 4
x y z

8 8 8

Caseii : 1
10
7

10

4
3
4

7
5
7

x 0
y 0

z 0

10 x 4 y 7 z 0
7 x 3 y 5z 0
10 x 4 y 7 z 0
x
y
z

4 7
10 7 10 4
3 5
7 5
7 3
x y z

1 1 2
Caseiii : 2
9 4 7
7 4 5

10 4 8
9x 4 y 7z 0

x 0
y 0

z 0

7 x 4 y 5z 0
10 x 4 y 8 z 0

DEPT OF MATHS, SJBIT

Page 156

ENGINEERING MATHEMATICS-I

15MAT11

x
y
z

4 5
7 5
7 4
4 8

10 8

10 4

x y z

12 6 12
1 1
p x1 x2 x3 0 1
1 2
4 2
p 1 0
3 1
0 0
p 1 AP 0 1
0 0
1

2
1
2

3
1
2

0
0 D
2
191 64
A5 PD 5 p 1 97 32
190 64

127
65
126

Quadratic forms:
A homogeneous expression of the second degree in any number of variables is
called a quadratic form (Q.F).
In general for two variables x1 , x2 i.e, n 1, 2 a11 x12 2a12 x1 x2 a22 x22
is called QF in
two variables.
1

2
coeff of x1 x2
coeff of x1
2
The matrix A of the above Q.F is A

1 coeff of x x coeff of x 2
1 2
1
2

Eg : x 2 y 2 xy

2) 2 x 2 3 y 2 6 xy
3) 5 x12 7 x22 12 x1 x2

DEPT OF MATHS, SJBIT

1
A
1
2
2
A
3

1
2

3
3

5
A
6

6
7

Page 157

ENGINEERING MATHEMATICS-I

15MAT11

Similarly Q.F in 3 variables is


a11 x12 a22 x22 a33 x32 2a12 x1 x2 2a13 x1 x2 2a23 x1 x2
1
1

2
coeff x1 x2
coeff x1 x3
coeff of x1
2
2

1
1 Coeff of x x coeff of x 2
coeff x2 x3
1 2
1
A 2
2
1
Coeff of x1 x2 1 coeff x2 x3 coeff of x32
2
2

Examples :1) QF : x 2 y 2 z 2 xy 2 yz 4 zx

2) 5 x 2 yz 6 y 9 z 4 xy
2

3) xy yz zx A

1
2

1
0
2
1 1
2 2

1
2

1
1
2
2 1

5 2 0
A 2 6 1
0 1 9

1
2

1
2

Canonical Form (sum of squares):


Q 1 y12 2 y22 ....... n yn2 where 1 , 2 ,.......n are Eigen values called canonical form.
Rank Index & Signature of canonical form

The number of non-zero terms present in a canonical form of Q is called rank of Q it, (r)
Ex :2 y12 y22 y32 r 3 y12 y32 r 2
1. The number of the terms present in a canonical form is called index of Q. (p)
Ex :2 y12 3 y22 5 y32 p 2
2. The difference between the negative terms in a canonical form is called signature
of Q (s)
DEPT OF MATHS, SJBIT

Page 158

ENGINEERING MATHEMATICS-I
Ex : y12 3 y22 y32

15MAT11

s 2 1 1

Nature of Quadratic Form:


r=rank, p=index n=number of variables
Condition
Meaning
Nature of Q.F
Eg;
r=n,p=n
All n Co efficient +ve definite
2 y12 y22 8 y32 +
are positive
r=n,p=0
All n Co efficient -ve definite
y12 y22 6 y32
are
-ve
r=p,p<n for r=2=p At least one of the +ve Semi
y22 5 y32
2<3
Co-efficient Zero definite
& all other Coefficientp are +ve
r<n,p=0
At least one of the -ve Semi definite y22 10 y32
Co-efficient Zero
& all other Coefficient are -ve
Note: Q.F is indefinite if some of the Co-efficient are +ve and some are -ve
Eg: y12 y22 3 y32
Working rule to reduce Q.F to Canonical (sum of squares) form by
orthogonal transformation.
1. Write down the matrix A to Q.F
2. Find the Eigen values & the corresponding eigen vectors of matrixA.

3.

Normalise the Eigen vector x1 , x2 , x3

i.e, x11

x1
x1

a
If x1 b x11 a 2 b 2 c 2
c
x
x
111iy x12 2
x31 3
x2
x3
4. Write down the associated orthogonal model matrix Q x11 x12 x31
5. Since pp1 I p 1 p1
Then p 1 AP p1 AP diagonal matrix 1 , 2 , 3
DEPT OF MATHS, SJBIT

Page 159

ENGINEERING MATHEMATICS-I

15MAT11

6. The associated canonical form is 1 y12 2 y22 3 y32

x1
y1


7. x=py where X x2 & Y y2
x3
y3
will give us the orthogonal linear transformation.
1) Obtain the canonical form of the quadratic form
Sol : 2 x 2 2 y 2 2 z 2 2 xy 2 yz 2 zx
2 1 1
A 1 2 1
1 1 2
(A I ) 0
2 1 1
1 2 1 0
1
1 2

(2 ) 4 4 1 2 1 3 0
2 4 3 3 3 0

(2 ) (2 ) 2 1 1(2 ) 1 11 (2 ) 0
2

2 2 8 6 3 4 2 3 3 3 0

3 6 2 9 0 3 6 2 9 0

( 2 6 9) 0 ( 3)3 0
0,3,3 i.e, 1 0, 2 0, 3 0 are roots and are the Eigen values of A.
2) The canonical form of the given Q.P that we get by an orthogonal transformation
is 1 y12 2 y22 3 y32 3 y22 3 y32
Sol: Since one Co- efficient in this canonical form is zero & the other two are +ve,
the Q.F is +ve Semi-definite
Rank, r=2 Index, p=2 & Signature, s=2.
3) Reduce the Q.F x12 3x22 3x32 2 x2 x3 to the canonical form by an orthogonal
transformation
1 0 0
Sol : A 0 3 1
0 1 3
DEPT OF MATHS, SJBIT

Page 160

ENGINEERING MATHEMATICS-I

15MAT11

(A I) 0
1

(1 ) 9 6 1 0
(1 ) 6 8 0

(1 ) (3 ) 2 1 0 0
2

2 6 8 3 6 2 8 0
3 7 2 14 8 0 3 7 2 14 8 0
( 1) ( 2) ( 4) 0
The eigen values of A are 1 1 2 2
Casei : For 1 1

3 4

0 0 0 x1 0
0 2 1 x 0

2
0 1 2 x3 0
x
y
z

2 1
0 1 0 2
1 2

0 1

x y z

3 0 0
3 1
x1 0 0
0 0
11x111 12 0 0 1

100 1 0
x
x 1

x1
1
T

1
1

2 2
1 0 0 x1 0
0 1 1 x 0

2
0 1 1 x3 0
x
y
z

1 1
1 0 1 0
1 1
0
1 0 1
DEPT OF MATHS, SJBIT

Page 161

ENGINEERING MATHEMATICS-I

15MAT11

x y z

0 1 1
3
x2 1
1
11x211 0 1 1 2
x
1
1
T
1
x2 2
0 1 1 0

x2
2
2
2

3 4
3 0 0 x1 0
0 1 1 x 0

2
0 1 1 x3 0
x
y
z

1 1
3 0 3 0
1 1
0 1 0 1
x y z

0
3 3
0 0
x1 3 1
3 1
11x311 0 1 1 2
x
0 1 1 1 1 1
x 3

x3
2
2
2

The orthogonal model matrix for A is

1
0
0

1
1

Q x11 x12 x31 0


2
2

1 1

0 2 2

X=Q.F is the orthogonal transformation that reduces the given Q.F to the canonical form.
The canonical form is
1 y12 2 y22 3 y32 y12 2 y22 4 y32
T

1
3

DEPT OF MATHS, SJBIT

Page 162

ENGINEERING MATHEMATICS-I

15MAT11

Rayleighs power method is an iterative method to determine the numerically largest


eigen value and the corresponding eigen vector of a square matrix.
Working procedure:
Suppose A is the given square matrix, we assume initially an eigen vector X 0 in a
simple for like [1,0,0] or [0,1,0] or [0,0,1] or [1,1,1] and find the matrix
product AX 0 which will also be a column matrix.
We take out the largest element as the common factor to obtain AX 0 1 X
1

We then find AX and again put in the form AX 2 X by normalization.


The iterative process is continued till two consecutive iterative values of and X
are same upto a desired degree of accuracy.
The values so obtained are respectively the largest eigen value and the
corresponding eigen vector of the given square matrix A.

Problems:
1) Using the Power method find the largest eigen value and the corresponding eigen
vector starting with the given initial vector.
2 0 1
0 2 0 given 1 0 0T

1 0 2

Solution : AX

AX 1

AX

2 0 1 1 2
1

0 2 0 0 0 2 0 1 X 1
1 0 2 0 1
0.5
2 0 1 1 2.5
1

0 2 0 0 0 2.5 0 2 X 2
1 0 2 0.5 2
0.8

2 0 1 1 2.8
1

0 2 0 0 0 2.8 0 3 X 3
1 0 2 0.8 2.6
0.93

DEPT OF MATHS, SJBIT

Page 163

ENGINEERING MATHEMATICS-I
2
AX 0
1
2
4
AX 0
1
2
5
AX 0
1

15MAT11

0 1 1 2.93
1

2 0 0 0 2.93 0 4 X 4
0.98
0 2 0.93 2.86
0 1 1 2.98
1

2 0 0 0 2.98 0 5 X 5
0.99
0 2 0.98 2.96
0 1 1 2.99
1

2 0 0 0 2.99 0 6 X 6
0.997
0 2 0.99 2.98

2 0 1 1 2.997
1
AX 0 2 0 0 0 2.997 0 7 X 7
1 0 2 0.997 2.994
0.999
Thus the largest eigen value is approximately 3 and the corresponding eigen vector is
[1 ,0, 1]
6

2) Using the Power method find the largest eigen value and the
corresponding eigen vector starting with the given initial vector.

4 1 1
2 3 1

2 1 5
4 1
0
Solution : AX 2 3
2 1
4
AX 2
2
4
2
AX 2
2
1

4
AX 2
2
4
4
AX 2
2
3

1 1
3 1
1 5
1 1
3 1
1 5

given 1 0.8 0.8

1
1
5

1 1
3 1
1 5
1 1
3 1
1 5

1 5.6
1
0.8 5.2 5.6 0.93 1 X 1

0.8 5.2
0.93
1 5.86
1
0.93 5.72 5.86 0.98 2 X 2

0.93 5.72
0.98

1 5.96
1
0.98 5.92 5.96 0.99 3 X 3

0.98 5.92
0.99
1 5.98
1
0.99 5.96 5.98 0.997 4 X 4

0.99 5.96
0.997
1 5.994
1
0.997 5.988 5.994 0.999 5 X 5

0.997 5.988
0.999

DEPT OF MATHS, SJBIT

Page 164

ENGINEERING MATHEMATICS-I

15MAT11

Thus after five iterations the numerically largest eigen value is 5.994 and corresponding
eigen vector is [1, 0.999, -0,999]
6) Using Rayleighs power method to find the largest Eigen value and the corresponding
Eigen vector of the matrix.
(Dec 2012)
6 2 2
X (0) 1,0,0
Sol: A 2 3 1
2 1 3

AX

(0 )

AX (1 )

6
2
2
6
2
2

2 2 1 6
1

3 1 0 2 6 0.333
0.3333
1 3 0 2
2 2 1
7.3332
1

3 1 0.333 3.3332 7.3332 0.4545


0.4545
1 3 0.3333 3.3332

6 2 2 1
7.818
1

AX 2 3 1 0.4545 3.818 7.818 0.488


2 1 3 0.4545 3.818
0.488
6 2 2 1
7.952
1

(3 )
AX 2 3 1 0.488 3.952 7.952 0.4969
2 1 3 0.488 3.952
0.4969
The largest Eigen value is 7.952 and the corresponding Eigen vector is
1 0.4969 0.4969
(2 )

DEPT OF MATHS, SJBIT

Page 165

Potrebbero piacerti anche